0% found this document useful (0 votes)
8 views38 pages

HY cvv

The document outlines key concepts related to heart failure, including the roles of endothelin 1 and nitric oxide in pulmonary hypertension, and differentiates between left and right heart failure symptoms. It discusses various heart conditions such as cor pulmonale, atrial septal defects, and ventricular septal defects, emphasizing their presentations and implications for treatment. Additionally, it highlights the importance of understanding murmurs and their associations with specific cardiac abnormalities for USMLE preparation.

Uploaded by

heran7989
Copyright
© © All Rights Reserved
We take content rights seriously. If you suspect this is your content, claim it here.
Available Formats
Download as PDF, TXT or read online on Scribd
0% found this document useful (0 votes)
8 views38 pages

HY cvv

The document outlines key concepts related to heart failure, including the roles of endothelin 1 and nitric oxide in pulmonary hypertension, and differentiates between left and right heart failure symptoms. It discusses various heart conditions such as cor pulmonale, atrial septal defects, and ventricular septal defects, emphasizing their presentations and implications for treatment. Additionally, it highlights the importance of understanding murmurs and their associations with specific cardiac abnormalities for USMLE preparation.

Uploaded by

heran7989
Copyright
© © All Rights Reserved
We take content rights seriously. If you suspect this is your content, claim it here.
Available Formats
Download as PDF, TXT or read online on Scribd
You are on page 1/ 38

MEHLMANMEDICAL.COM MEHLMANMEDICAL.

COM

IM Cardio - Endothelin 1 is vasoconstrictor and key mediator in pulmonary hypertension.


USMLE wants you to know this is increased in both cor pulmonale and left heart
failure. Bosentin is an endothelin 1 receptor antagonist.
Important basic heart failure points - Nitric oxide synthase, in contrast, USMLE wants you to know is decreased in
- Presents as pulmonary findings (i.e., dyspnea, orthopnea, paroxysmal pulmonary hypertension (makes sense, since NO dilates).
nocturnal dyspnea). - A loud P2 and tricuspid regurgitation are HY findings in cor pulmonale. I
- This is because left-heart problems cause a backup of pressure onto the discuss these in more detail in the tables below.
pulmonary circulation, leading to increased pulmonary capillary hydrostatic
pressure à transudation of fluid into the alveolar spaces (pulmonary edema).
Sometimes this can also cause pleural effusion.
- Left atrial pressure (LAP) = pulmonary capillary wedge pressure (PCWP). Hyper-quick causes of bilateral pitting peripheral edema
Left heart failure - Therefore, if there is left heart pathology, PCWP is high (exceedingly HY). - Right heart failure (either due to cor pulmonale or congestive) à ¯ ability to fill right
- Conversely, if a Q gives you normal PCWP, you know there’s nothing wrong heart à ­ central venous pressure à ­ systemic venous hydrostatic pressure à
with the left heart. Qs will often give high PCWP and low BP, where you need to transudation of fluid from systemic veins/venules into interstitium of legs.
know immediately that means cardiogenic shock. - Answer on NBME for why HTN doesn’t automatically cause peripheral edema à
- What USMLE will do is give you some sort of left-heart pathology + dyspnea, Cardiac answer = “increased pre-capillary resistance.” In other words, arterioles are responsible
and then ask for the cause of the dyspnea à answer = “increased pulmonary for the majority of peripheral resistance; in the setting of high BP, the reason capillary
capillary hydrostatic pressure.” Another answer in this case is “increased hydrostatic pressure isn’t automatically high enough where the transudation threshold is
alveolar-arteriolar (A-a) oxygen gradient.” reached is because arterioles constrict, thereby ­­ resistance and reducing excessive
- Presents as systemic findings – i.e., jugular venous distension (JVD) and blood flow through the capillaries.
peripheral edema. - Cirrhosis à ¯ hepatic production of albumin à ¯ intravascular oncotic pressure à
- Since blood cannot enter the right heart as easily, it backs up to the neck veins Hepatic
transudation of fluid from systemic veins/venules into interstitium of legs.
(JVD) and venous circulation (increased hydrostatic pressure in veins à - Proteinuria à hypoalbuminemia à ¯ intravascular oncotic pressure à transudation
transudation of fluid into legs). The Q might mention that central venous Nephrogenic
of fluid from systemic veins/venules into interstitium of legs.
Right heart failure pressure is high. - Dihydropyridine calcium channel blockers (i.e., amlodipine, nifedipine).
- Hepatosplenomegaly can also be seen in RHF but is very rare on USMLE. Drugs - Imatinib (targets BCR/ABL tyrosine kinase in CML).
- Normal jugular venous pressure (JVP) is 3cm above the sternal angle. JVD - Miscellaneous mechanisms not important for USMLE. Just know above drugs do it.
would be higher than this. Sometimes questions can write that jugular venous
- Strict vegetarianism or veganism à ¯ dietary protein consumption à ¯ intravascular
pulsations are seen 3cm above the sternal angle and the student erroneously
Dietary oncotic pressure à transudation of fluid from systemic veins/venules into interstitium
thinks this means JVD, but this is not the case.
of legs.
- Congestive heart failure = left heart failure + right heart failure.
Pregnancy - A little bit of peripheral edema is normal in pregnancy due to compression of IVC.
- The most common cause of right heart failure is left heart failure. Simply
adding the two together, we now call that congestive heart failure.
Congestive heart failure
- In congestive heart failure, we’ll see both left- and right-heart failure findings –
i.e., patient will have dyspnea, JVD, and peripheral edema.
Hyper-quick causes of unilateral non-pitting edema
- PCWP is elevated in these patients, since the left heart has pathology.
- Malignancy (e.g., peau d’orange of breast), Hx of surgery (e.g., mastectomy),
- Cor pulmonale is defined as right-heart failure due to a pulmonary cause. In Lymphatic insufficiency
Wuchereria bancrofti (elephantiasis).
other words, the left heart is completely normal in cor pulmonale and PCWP is
normal. - Pretibial myxedema (Graves) à mucopolysaccharide deposition in skin +
surrounding edema.
- Cor pulmonale will be a patient who has JVD and peripheral edema in the
- Myxedma (severe hypothyroidism) à despite the name, it refers to general
setting of obvious and overt lung disease, such as 100-pack-year smoking
“severe hypothyroidism,” not just skin changes; can cause carpal tunnel
history, cystic fibrosis, or pulmonary fibrosis. These can present with lung Thyroid
findings such as wheezes, where you as the student need to say, “It just doesn’t syndrome.
seem like they’re focusing on left-heart failure as the cause of the right-heart - “Pretibial myxedema” is only seen in Graves. Paradoxical hyperthyroidism seen
in Hashimoto causing pretibial myxedema is astronomically rare and will get you
failure here. It seems the 100-pack-year smoking Hx causing COPD is why the
Cor pulmonale questions wrong on USMLE.
right heart is failing.”
- The patient can have a “boot-shaped” heart colloquially, which refers to right
ventricular hypertrophy without left ventricular hypertrophy.
- If the patient has COPD, the massively hyperinflated lungs will push the heart
to the midline, causing a long, narrow cardiac silhouette, with a point of HY Valvular / flow abnormalities on USMLE
maximal impulse in the sub-xiphoid space. - Fixed splitting of S2.
- You must know that pulmonary hypertension is the reason the right heart - Can sometimes be associated with a systolic flow murmur, since more
decompensates. In both cor pulmonale and congestive heart failure, the right blood L à R from the LA à RA means more blood flow across the
heart experiences increased afterload because of pulmonary hypertension. Atrial septal defect pulmonic valve. So Q might say “fixed splitting of S2 and a systolic
murmur.”
- Sometimes can be seen in Qs as “wide, fixed splitting.” I only mention
this because some students get pedantic / ask about this. “Wide splitting”

MEHLMANMEDICAL.COM 4 MEHLMANMEDICAL.COM 5
MEHLMANMEDICAL.COM MEHLMANMEDICAL.COM

just means right ventricular hypertrophy. So if the Q says “wide, fixed


splitting,” they’re saying the patient has RVH due to an ASD.
- Patent foramen ovale = ASD on USMLE. Don’t confuse with patent ductus
arteriosus (discussed below).
- USMLE loves asking questions (on 2CK, yes) showing you change in
oxygen in the chambers of the heart and making you choose ASD, VSD, etc.

- NBME loves this style of Q. You can see O2 somehow increased from RA
to RV. The only way this is possible is if we have a VSD where oxygenated
blood moves from LV à RV.

- For example, you can see above that somehow O2 increases from the
SVC to the RA, which is ordinarily impossible. The only way this could occur
is if an ASD is present, where oxygenated blood moved from LA à RA.
- ASDs can sometimes be responsible for “paradoxical emboli,” where a
DVT leads to stroke. This is ordinarily impossible, since a clot embolizing to
the lungs via the venous circulation has no way of reaching the arterial
circulation. But if an ASD is present, the clot can go RA à LA à LV à up to
the brain, causing stroke.
- ASDs do not need to be repaired unless patient has evidence of
pulmonary hypertension, RVH, arrhythmia (usually AF), or paradoxical
embolus.
- Holosystolic (aka pan-systolic) murmur at lower left sternal border.
- Can be associated with a diastolic rumble or enlarged left atrium (if more
blood going L à R across VSD, then more blood is returning to the LA from
the lungs à LA dilatation). - This one might initially appear a little more difficult. This is Eisenmenger
- Seen as part of tetralogy of Fallot (VSD, RVH, overriding aorta, pulmonic syndrome, where we have a reversal of flow from RV à LV across the VSD.
stenosis). The NBME is known to show this diagram as well.
- If you’re wondering why oxygen % goes from 99 to 96 from the
- If a VSD is repaired, USMLE wants ­ LV pressure, ¯ RV pressure, and ¯ LA
pulmonary circulation to the LA, this is because of thebesian veins draining
pressure as the changes now seen in the heart.
the myocardium itself, which open into the different heart chambers,
- VSD does not cause cyanosis at birth. Only years later after the higher
including the LA. If you think that’s weird, take it up with NBME, not me,
blood flow to the lungs results in pulmonary hypertension, followed by
since they have the 99 to 96 drop-down on their diagrams.
right ventricular hypertrophy and reversal R à L (Eisenmenger) does the
- VSDs are repaired if patient develops pulmonary hypertension, RVH,
patient become cyanotic.
arrhythmia, Eisenmenger syndrome, recurrent endocarditis (turbulence of
Ventricular septal defect - Murmur can be silent or soft at birth, followed by loud at 7 days of life.
blood due to VSD can ­ risk of valve infections), or aortic regurgitation (if
The USMLE will ask why the murmur is louder now à answer = decreased
pulmonary vascular resistance – i.e., the lungs open up during the first VSD located near the aortic valve).
week of life, resulting in decreased RV pressure and an increase in the L à - Seen in Down syndrome.
R pressure gradient (louder murmur). Atrioventricular septal defect - Between the atrium and ventricle, aka “endocardial cushion defect,”
- Conversely, if they ask why the murmur was softer at birth compared to although this latter term can also apply to ASD and VSD in Downs.
now, the answer is “increased pulmonary vascular resistance,” where the - Holosystolic (pan-systolic) or just regular “systolic,” 29 times out of 30.
lungs were still closed at the time, so there was a lesser gradient L à R - Most USMLE questions will not mention it radiating to the axilla.
(softer murmur). - Highest yield cause of MR on USMLE is post-MI papillary muscle rupture.
- Similar to ASD Qs, USMLE loves giving you diagrams with changes in O2 Mitral regurgitation USMLE is obsessed with this. They’ll say hours to days after an MI, patient
between the chambers and then making you infer we have a VSD. Yes, this has new-onset systolic murmur à answer = MR.
stuff is asked on 2CK. - Seen acutely in rheumatic heart disease (valve scars over years later and
becomes mitral stenosis).

MEHLMANMEDICAL.COM 6 MEHLMANMEDICAL.COM 7
MEHLMANMEDICAL.COM MEHLMANMEDICAL.COM

- Can be caused by general ischemia / dilated cardiomyopathy. - USMLE loves using MVP as a distractor in panic disorder questions. They
- Can cause JVD (i.e., back up all the way to the right heart); this is asked will give long paragraph about panic attack/disorder + also mention there’s
multiple times on the new NBMEs. a mid-systolic click; they’ll ask for cause of patient’s presentation à
- You do not do preoperative stress tests to determine perioperative MI answer = panic disorder, not MVP à student is confused because they say
risk if the patient has mere mitral regurg without other risk factors. For mid-systolic click, but the MVP isn’t the cause of the patient’s
example, one of the 2CK forms gives a Q where smoker with MR has no presentation; the panic disorder is; MVP’s are usually incidental, benign,
shortness of breath or chest pain with exertion, and the answer is “no and asymptomatic.
further management indicated,” where exercise stress test is wrong. - MVP does not progress to mitral regurg almost always. So don’t think
- If the patient has Sx of heart failure or ischemia, then we do pre-op stress that MVP and MR are the same.
test to determine MI risk. I discuss stress tests later in this chapter. - Decrescendo holo-diastolic (pan-diastolic) murmur; can also be described
- The mitral valve is replaced if the patient develops severe pulmonary as “early diastolic murmur,” or “diastolic murmur loudest after S2.”
symptoms (i.e., shortness of breath / reduced exercise tolerance), reduced - Causes wide pulse pressure (i.e., big difference between systolic and
ejection fraction, arrhythmia, or endocarditis if valve function is destroyed. diastolic pressures, e.g., 160/50, or 120/40) à results in head-bobbing and
- Described as “rumbling diastolic murmur with an opening snap”; can also bounding pulses (don’t confuse with slow-rising pulses of aortic stenosis).
be described as “decrescendo mid-late diastolic murmur” (i.e., following - The bounding pulses can be described on NBME as “brisk upstroke with
the opening snap). precipitous downstroke.” In turn, they can just simply say, “the pulses are
- Can cause a right-sided S4 if the pressure backs up all the way to the right brisk,” meaning the systolic component is strong.
heart (seen on NBMEs sometimes; this confuses students because they - I would say 4/5 times bounding pulses means AR. The other 1/5 will be
think S4 must be LV, but it’s not the case). An S4 is a diastolic sound heard PDA and AV fistulae (discussed below). Bounding pulses occur when blood
in either the LV or RV when there is diastolic stiffening due to high Aortic regurgitation quickly leaves the arterial circulation. In AR, the blood quickly collapses out
afterload. of the aorta back into the LV. In PDA, it leaves the aorta and enters the
- 99% of mitral stenoses are due to Hx of rheumatic heart disease (i.e., the ductus arteriosus; in AV fistulae, it leaves for a vein.
patient had rheumatic fever as a child, where at the time it was mitral - Highest yield cause on USMLE is aortic dissection à can retrograde
regurg, but years later it has now become mitral stenosis). propagate toward the aortic root causing aortic root dilatation and AR.
- One 2CK NBME Q mentions patient with history of rheumatic heart - Even though MVP is most common in Marfan and Ehlers-Danlos, AR is
disease who, years later, now has 4/6 rumbling diastolic murmur without second most common in these patients, since if they get aortic dissection,
an opening snap; this is still mitral stenosis. Although opening snap is this can lead to AR.
buzzy for MS, just be aware it’s not mandatory and that this Q exists on - Can lead to volume overload on the LV and eccentric hypertrophy.
NBME. - Valve is replaced if patient has EF <50%, there is significant left
- Other HY presentation on USMLE is pregnant women with new-onset ventricular dilatation, or if severe endocarditis has obliterated the valve.
Mitral stenosis
dyspnea in 2nd trimester and a diastolic murmur. This is because 50% - Mid-systolic murmur, or just “systolic” murmur; can also be described as
increase in plasma volume by 2nd trimester causes the underlying “late-peaking systolic murmur with an ejection click.”
subclinical MS to become symptomatic. Don’t confuse this with severe - Radiates to the carotids. This descriptor shows up quite frequently on
dyspnea and peripheral edema in late third-trimester, which is instead NBME (way more than radiation to the axilla for MR).
peripartum cardiomyopathy (antibody-mediated). - Causes slow-rising pulses, aka “pulsus parvus et tardus” (don’t confuse
- The 1% of MS that’s not due to Hx of RF can be marantic (non-bacterial with bounding pulses of AR).
thrombotic endocarditis; NBTE) à endocarditis seen due to - SAD à Syncope, Angina, Dyspnea; classic combination seen in AS, albeit
hypercoagulable state in the setting of malignancy, where the vegetations not mandatory. If you get a question where they say systolic murmur but
are small and verrucous, on both sides of the valve. This is in contrast to you’re not sure of the diagnosis, if they say chest pain or fainting, you
Aortic stenosis
bacterial endocarditis, which causes large, floppy vegetations that lead to know it’s AS.
MR, not MS. - Often caused by bicuspid aortic valve. The patient need not have Turner
- Libman-Sacks endocarditis seen in SLE is due to antiphospholipid syndrome and often won’t. Bicuspid valve is usually inherited as an
antibodies and is a type of NBTE. autosomal dominant familial condition.
- Balloon valvuloplasty is the 1st-line Tx for mitral stenosis. This is done if - The bicuspid valve need not calcify in middle-age prior to the AS forming.
patient has minimal calcification of the valve + has pulmonary HTN. Bicuspid valve can present with AS murmur in child or high schooler.
- Mitral valve replacement is done if balloon valvuloplasty fails, if patient - Do aortic valve replacement on 2CK if 1) cross-section of valve is <1.0
has severe MS with dyspnea, arrhythmia, or calcification of the valve. cm2, or 2) there is SAD. They ask both of these as separate Qs where they
- Most common murmur. want valve replacement.
- Described as mid-systolic click. - Will be described on USMLE as a holosystolic murmur that increases with
- “Myxomatous degeneration” is buzzy term that refers to connective inspiration.
tissue degeneration causing MVP in Marfan and Ehlers-Danlos. - Right-sided heart murmurs get worse with inspiration à diaphragm
Mitral valve prolapse - Almost always asymptomatic. On 2CK forms, they want you to know moves down à decreased intra-thoracic pressure à increased right-heart
Tricuspid regurgitation
about “mitral valve prolapse syndrome,” which is symptomatic MVP that filling.
presents as repeated episodes of “fleeting chest pain” on the left side in an - Can cause pulsatile liver.
otherwise healthy patient 20s-30s. They might say there is Hx of MI in the - Highest yield cause of TR on USMLE is pulmonary hypertension / cor
family, but this is MVPS, not MI. Answer on NBME is “no Tx necessary." pulmonale. I see this all over the NBME exams. For whatever reason, these

MEHLMANMEDICAL.COM 8 MEHLMANMEDICAL.COM 9
MEHLMANMEDICAL.COM MEHLMANMEDICAL.COM

conditions do not cause pulmonic regurg; they cause tricuspid regurg. In - Indomethacin (NSAID) will close the PDA.
other words, if you see tricuspid regurg in a Q, your first thought should be - Prostaglandin E1 is used to keep a PDA open (if a kid with congenital
pulmonary hypertension or cor pulmonale (right heart failure due to a heart malformations is born cyanotic and we need to buy time until
pulmonary cause). surgery).
- IV drug user endocarditis is obvious risk factor for TR, but weirdly - An open PDA can mask cyanosis in a newborn in a variety of conditions
nonexistent on USMLE. (i.e.,., hypoplastic left heart syndrome or pre-ductal coarctation). If they
- Carcinoid syndrome is theoretical cause but lower yield. tell you a kid is born with normal APGAR scores but a week later becomes
- Nonexistent murmur on USMLE. I don’t think I’ve ever seen this assessed cyanotic and they ask why, the answer is “closure of ductus arteriosus.”
once on any NBME exam for Steps 1 and 2 combined. - 1) Pulmonic stenosis; 2) RVH; 3) overriding aorta; 4) VSD.
Tricuspid stenosis - In theory, would be a rumbling diastolic murmur similar to mitral - If you’re asked which component most determines prognosis, the answer
stenosis, but would increase with inspiration since it’s on the right side of is the degree of pulmonic stenosis.
the heart. - The child will not be cyanotic at birth, but then years later, will develop
- Same as with tricuspid stenosis, this is a nonexistent murmur on USMLE. Eisenmenger syndrome (i.e., a reversal of the LàR shunt over the VSD to
I’ve never seen it assessed. be RàL) and cyanosis, where the stem gives a school-age kid who squats
Pulmonic regurgitation Tetralogy of Fallot
In theory it would be the same as aortic regurg but on the right (i.e., holo- on the playground to relieve symptoms.
diastolic murmur), but increases with inspiration. - Squatting ­ afterload, which ­ LV pressure, which ¯ the pressure
- Seen in tetralogy of Fallot. gradient of the RàL shunt, thereby mitigating cyanosis.
- Described as mid-systolic murmur, or just regular “systolic” murmur, that - Squatting also ­ preload by ­ venous return back to the right heart. But it
increases with inspiration, at the left sternal border, 2nd intercostal space. is the effect of ­ afterload that is most related to the ¯ in symptoms.
Pulmonic stenosis
This is the theoretical location, whereas AS is the 2nd intercostal space on - Tx is surgical correction in infancy or early childhood.
the right, not left. But the USMLE often isn’t strict about murmur locations - Systolic murmur seen in the setting of higher heart rate caused by
this way. infection, anemia, or pregnancy. Caused by increased flow across the
- Ductus arteriosus is special vessel in fetal circulation that connects the pulmonic and/or aortic valves.
proximal pulmonary trunk to the descending arch of the aorta. This allows - Known as a functional murmur because this means it goes away once the
for blood to bypass the high-resistance lungs in utero. After birth, this heart rate comes back down.
vessel should close, resulting in a remnant called the ligamentum Functional (flow) murmur - Seen all over 2CK forms for kids, where they try to trick you into thinking
arteriosum, but sometimes it does not close à PDA. the kid has a valvular pathology of some kind, but there isn’t; there will
- If a PDA occurs, blood moves in the neonate LàR (i.e., opposite of in merely be an infection or simple viral infection.
utero) from the descending arch of the aorta to the pulmonary trunk. - Can be seen sometimes with ASD, where the patient will have fixed
splitting of S2 “plus a systolic murmur” à merely higher right-sided
volume, so more flow across the pulmonic valve.
- On 2CK form; described as a murmur in the neck that abates when the
Venous hum kid is laid supine + the neck rotated.
- Benign + don’t treat.
- Associated with cardiac tumors (i.e., myxoma in adult, or rhabdomyoma
in kids for tuberous sclerosis).
“Ball-in-valve” murmur
- Described as a diastolic rumbling murmur that abates when the patient is
re-positioned unconventionally (e.g., onto his or her right side).
Patent ductus arteriosus - Aka persistent fetal circulation.
(PDA) - Q will give a post-term birth at 42 or 43 weeks + meconium-stained fluid
+ echo of the neonate shows a RàL shunt across the foramen ovale.
Student says, “Wait, but isn’t the foramen ovale between the atria, and
Persistent fetal hypertension that’s only open in the fetus but is supposed to close after birth?” Correct.
Hence we have persistent fetal circulation.
- Answer on USMLE will be “failure of pulmonary vasodilation.” Meconium
- You can see in the above diagram, somehow the blood become more aspiration syndrome can ¯ opening of the lung vasculature, leading to ­
oxygenated from the RV to the pulmonary artery, which is ordinarily right heart pressure and ­ risk of persistent fetal circulation.
impossible. The only way this could have occurred is if oxygenated blood - Both are diastolic sounds.
came LàR from the aorta to the pulmonary artery via a PDA. - S3 is due to high volume/preload in the left ventricle, causing a
- Murmur described three ways on USMLE: 1) continuous, machinery-like reverberation against the wall.
murmur; 2) pan-systolic pan-diastolic murmur (meaning it’s continuous - S3 can sometimes be physiologic (i.e., normal / no problem) in pregnancy
S3 versus S4
throughout both systolic and diastole); and 3) to-and-fro. The latter shows and high-endurance athletes. Patient will have eccentric hypertrophy
up on 2CK offline NBME 6. (sarcomeres laid in linear sequence). If pathologic, it is due to dilated
- Classically associated with congenital rubella (HY). They’ll give a kid born cardiomyopathy with reduced ejection fraction (<55%) or high-output
with a PDA and then ask what the mom experienced while pregnant; cardiac failure (EF >70%).
answer = arthritis and/or rash (rubella often presents as arthritis in adults).

MEHLMANMEDICAL.COM 10 MEHLMANMEDICAL.COM 11
MEHLMANMEDICAL.COM MEHLMANMEDICAL.COM

- There is one question on IM CMS form 7 where they give an S3 in - USMLE doesn’t give a fuck about pre- vs post-ductal. Pre-ductal in theory
diastolic dysfunction. I’m convinced this is an erratum, but I need to will be a very sick neonate. Post-ductal will be an adult (most cases).
mention it because it exists on the NBME form. - The vertebral artery (goes to brain) is the first branch of the subclavian
- S4 is due to high pressure/afterload on the left (but sometimes right) artery (goes to arm).
ventricle, causing a stiffened ventricle with diastolic dysfunction and - If there is a narrowing/stenosis of the proximal subclavian prior to the
concentric hypertrophy (sarcomeres laid in parallel). It is always branch point of the vertebral artery, this can lead to lower pressure in the
pathologic. It is usually caused by systemic hypertension causing afterload vertebral artery.
on the LV, or aortic stenosis. - This can cause a backflow of blood in the vertebral artery, producing
- S4 can sometimes be right-sided on USMLE. There is a 2CK Q where they miscellaneous neuro findings such as dizziness.
give severe mitral stenosis and say there’s an S4, but it’s for the RV not LV. - Blood pressure is different between the two arms.
Some weird/annoying points: Subclavian steal syndrome - USMLE will ask the Q one of two ways: 1) they’ll give you dizziness in
- The combo of S3 and S4, seen together in the same vignette, can be seen someone who has BP different between the arms and then ask for merely
in high-output cardiac failure. For example, they will say a patient as an AV “subclavian steal syndrome,” or “backflow in a vertebral artery” as the
fistula/conduit, or has Paget disease, and they will say there’s S3 and S4 answer. Or 2) they’ll give you BP in one of the arms + give you dizziness,
and ask for diagnosis à answer = high-output cardiac failure. The take- then the answer will be, “Check blood pressure in other arm.”
home point is that high-output failure can present with either an isolated - Next best step in Dx is CT or MR angiography (asked on 2CK NBME).
S3 or the combo of S3 and S4 together, but never S4 alone on USMLE. - I should point out that probably 3/4 questions on USMLE where blood
- One of the highest yield cardiac sounds on USMLE, almost always pressure is different between the arms, this refers to aortic dissection. But
overlooked by students. 1/4 is subclavian steal syndrome. As per my observation.
- Means pulmonary hypertension or cor pulmonale on USMLE. - Presents same as subclavian steal syndrome with otherwise unexplained
- The pulmonic valve slams shut due to high pressure distal to it. dizziness, but blood pressure is not different between the arms because
- For example, they’ll give a smoker who simply has a loud P2 à this just the subclavian is not affected.
Vertebral artery stenosis
means patient has pulmonary hypertension. Not complicated. - Caused by atherosclerosis. CT or MR angiography can diagnose.
Loud P2 - Also recall that I said above that highest yield cause of tricuspid regurg on - “Vertebrobasilar insufficiency” is a broader term that refers to patients
USMLE is pulmonary hypertension / cor pulmonale. So both what I want who have either subclavian steal syndrome or vertebral artery stenosis.
you to remember is both TR and loud P2 for this. - 2CK forms assess vertebral artery dissection, where they want you to
- Sometimes the UMSLE will just say “loud pulmonic component of S2,” or know a false lumen created by dissection in a vertebral artery can lead to
“loud S2,” rather than saying “loud P2.” I’ve never seen “loud A2” on stasis and clot formation, which in turn can embolize to the brain and
USMLE, but in theory this means systemic hypertension. cause stroke.
- A soft P2 refers to pulmonic stenosis, but is LY. Vertebral artery dissection - NBME can mention recent visit to a chiropractor (neck manipulation is
- Means right ventricular hypertrophy on USMLE. known cause).
- A2 and P2 are far apart. - The answer on the NBME is heparin for patients who have experienced
- You don’t have to worry about the mechanism. But in short, the more posterior stroke due to vertebral artery dissection. Sounds weird because
Wide splitting of S2 pressure you have in a ventricle, the more delayed the semilunar valve will it’s arterial, but it’s what USMLE wants.
close. So if we have RVH, P2 occurs later, widening the split. - Shows up on 2CK form as patient with stroke-like presentation + who
- Wide splitting of S2, right-axis deviation on ECG, and right bundle branch simultaneously has ipsilateral facial/neck pain.
block (RBBB) all = right ventricular hypertrophy on USMLE. Carotid artery dissection - The pain is due to stretching of nociceptors secondary to vascular
- Means left ventricular hypertrophy on USMLE. dilation.
- A2 occurs after P2 (normally we have A2 before P2). - Stasis within false lumen can lead to embolus to brain/eye.
- Left ventricular pressure is high and A2 delayed to the point that it - Caused by atherosclerosis.
Paradoxical splitting of S2
actually occurs on the opposite side of P2. - HTN biggest risk factor for atherosclerosis specifically of the carotids
- Paradoxical splitting of S2, left-axis deviation on ECG, and left bundle (strong systolic impulse pounds the carotids à endothelial damage à
branch block (LBBB) all = left ventricular hypertrophy on USMLE. atheromatous plaque formation).
- Refers to narrowing of the aortic arch (this is referred to as coarctation; - Carotid bruit only seen in about 25% of Qs. Don’t rely on this as crutch.
do not use the word stenosis to describe this). - Vignette will give a stroke, TIA, or retinal artery occlusion in the setting of
- Classically seen in Turner syndrome, but absolutely not mandatory. a patient with HTN. à You have to be able to make the association that a
Shows up idiopathically in plenty of NBME Qs. I point this out because plaque from one of the carotids has launched off, since HTN = ­ risk.
students often think the patient must have Turner syndrome. Carotid artery stenosis - USMLE will then ask for management (2CK only):
- Presents as upper extremities that have higher BP, brisk pulses, and are - Do carotid duplex ultrasonography as next best step in diagnosis to look
Coarctation of the aorta warmer; the lower extremities have lower BP, weak pulses, and are cooler. for degree of occlusion. I’ve never seen carotid angiography as a correct
- Sometimes the Q can just say, “the radial pulses are brisk.” à The answer on NBME exams.
implication is, “Well if they’re saying specifically that the radial pulses are - If occlusion >70% symptomatic, or >80% asymptomatic, then do
brisk, that must mean the pulses in the legs aren’t.” endarterectomy. “Symptomatic” = stroke, TIA, or retinal artery occlusion.
- Murmur sound not important for USMLE. Can sometimes be described as A mere bruit is not a symptom; that is a sign.
a systolic murmur heart in the infrascapular region. - If under these thresholds, do medical management only, which requires a
- Can cause LVH with left-axis deviation ECG. triad of: 1) statin; 2) ACEi or ARB; and 3) anti-platelet therapy.

MEHLMANMEDICAL.COM 12 MEHLMANMEDICAL.COM 13
MEHLMANMEDICAL.COM MEHLMANMEDICAL.COM

- The USMLE will not force you to choose between low- and high-potency
statins.
- USMLE tends to list lisinopril as their favorite ACEi for HTN control.
- It’s to my observation aspirin alone is sufficient on NBME exams for anti-
platelet therapy, even though in real life patient can receive either aspirin
alone; the combo of aspirin + dipyridamole; or clopidogrel alone.
- USMLE will not give borderline carotid occlusion thresholds – i.e., they’ll
say either 30% or 90%. If they list the % as low, look at the vignette for the
drugs they list the patient on. Sometimes they’ll show the patient is
already on statin, lisinopril, and aspirin, and then the answer is just
“continue current regimen.” I have once seen “add clopidogrel” as a wrong
answer in this setting, which makes sense, since the combo of aspirin +
clopidogrel is never given anyway.
- Sometimes they will give you a low carotid occlusion % + say the patient
is on 2 of 3 drugs in the triad, and then the answer is just “add aspirin,” or
“add statin,” or “add lisinopril.”
- If the vignette doesn’t mention elevated BP but says you have some
random dude over 50 with a stroke, TIA, or retinal artery occlusion, the
- You do not need to memorize these aortic aneurysm types. I’m just
next best step is carotid ultrasonography to look for carotid stenosis. In
showing you that if the common iliacs are involved (as with left image), BP
other words, it is assumed the patient has a carotid plaque in this setting.
can differ as well between the legs. For 2CK:
- If the vignette gives patient with episodes of unexplained syncope or
- Tx for ascending aortic aneurysm (type A) = labetalol + surgery.
light-headedness, but not stroke, TIA, or retinal artery occlusion, then the
- Tx for descending aortic aneurysm (type B) = labetalol alone initially.
next best step is ECG, followed by Holter monitor, looking for atrial
- Caused by deceleration injury. Most common cause of death due to car
fibrillation (AF causes LA mural thrombus that launched off to brain/eye).
accident or fall. Exceedingly HY on 2CK.
- The triad of 1) statin; 2) ACEi or ARB; and 3) anti-platelet therapy is also
- Will be described as patient following an MVA who has “widening of the
done for general peripheral vascular disease unrelated to carotid stenosis
mediastinum.” They’ll then ask for the next best step à answer = aortic
(i.e., if a patient has intermittent claudication).
angiography (aka aortography), OR CT angiography.
- Stroke, TIA, or retinal artery occlusion, if they don’t mention HTN, but
- New 2CK form has “CT scan of the chest” straight up as the answer,
they mention an abdominal bruit, you will still do a carotid duplex
which refers to CT angiography. NBME/USMLE will not force you to choose
ultrasound. The implication is that the bruit in the abdomen could be a
between aortography or CT angiography; they’ll just list one.
AAA or RAS, where atherosclerosis in one location means atherosclerosis Traumatic rupture
- Labetalol used first-line in patients who have aortic dissection and
everywhere, so the patient likely has carotid stenosis by extension. They of the aorta
traumatic rupture of the aorta. Nitroprusside comes after.
once again need not mention carotid bruit; apparently it is not a sensitive
- Labetalol is answer on NBME even in patient who has low BP due to
finding (i.e., we cannot rule-out ­ occlusion just because we don’t hear it).
rupture or dissection due to the drug ¯ shearing forces. I’ve seen students
- Classically presents as severe upper chest pain radiating to the back
get this wrong saying, “But patient has low BP though.” My response is, file
between the scapulae.
a complaint with the exam not with me.
- As discussed above in the aortic regurg section, USMLE loves this as most
- 2CK Q gives “esmolol + nitroprusside” as answer to a traumatic rupture
common cause of AR due to retrograde propagation toward the aortic
Q, but almost always, they will just want “labetalol.”
root. For example, patient with Hx of HTN, cocaine use, or a connective
- Emergency surgical repair is indicated following IV drug administration.
tissue disorder (i.e., Marfan, Ehlers-Danlos) who has a diastolic murmur,
- Can present as “visible pulsation” on USMLE.
you should be thinking immediately that this is dissection.
- For aortic aneurysm, they can say “visible pulsation above the
- “Medial necrosis” is a term that is used on NBME exams to describe
manubrium,” or “pulsatile mass above the manubrium.” There can also be
changes to the aorta in dissection. In the past, “cystic medial necrosis”
a tracheal shift. I’ve seen students select pneumothorax here. But for
Aortic dissection used to be buzzy for dissection due to Marfan syndrome, but I haven’t
whatever reason you can get tracheal shift in thoracic aortic aneurysm. For
seen USMLE care about this. I have, however, seen a dissection Q on
AAA, there can be “visible pulsation in the epigastrium.”
NBME where it is due to hypertension, and simply “medial necrosis” is the
- Biggest risk factor for AAA is smoking.
answer.
- Do a one-off abdominal ultrasound in both men and women 65+ who are
- As mentioned above, 3/4 Qs where BP is different between the arms
Aortic aneurysm ever-smokers. This screening used to be just performed on men, but now
refers to aortic dissection. A Q on 2CK IM form 7 has “thoracic aortic
it includes women.
dissection” where not only is the BP different between the arms, but it’s
- AAA repair is indicated if the aneurysm is >5.5 cm or the rate of change of
also different between the L and R legs (i.e., L-leg BP is different from R-leg
size increase is >0.5cm/month for 6 months. This is on 2CK form, where
BP) à sometimes thoracic aortic dissections can anterograde propagate
they give a patient with a 4-cm AAA and ask why serial ultrasounds are
all the way down to the abdominal aorta.
indicated à answer = “size of aneurysm.”
- In general, perioperative MI risk is assessed using a pre-op stress test.
2CK NBME Q has dipyridamole and thallium pharmacologic stress test as
answer in patient with 6-cm AAA prior to surgery.

MEHLMANMEDICAL.COM 14 MEHLMANMEDICAL.COM 15
MEHLMANMEDICAL.COM MEHLMANMEDICAL.COM

- Diabetes is protective against aneurysm. Non-enzymatic glycosylation of


endothelium causes stiffening of the vascular wall.
- Don’t do AAA repair on USMLE in patient who has advanced
comorbidities or terminal disease, e.g., stage 4 lung cancer.
- Tangential: 2CK loves “pulsatile hematoma” in the neck in trauma
patients, where the answer is “endotracheal intubation.” Sounds nitpicky,
but shows up repeatedly.
- Can be idiopathic, iatrogenic (i.e., dialysis), from injury (i.e., stab wound),
or caused by other disease (i.e., hereditary hemorrhagic telangiectasia or
Paget disease of bone).
- Similar to aortic aneurysms, AV fistulae can sometimes present with
pulsatile mass, but in a weird location, e.g., around the left ear in patient
with tinnitus (on NBME exam). Student says, “Why is it at the left ear
though?” à No fucking idea. Take it up with NBME.
- Highest yield point is they can cause high-output cardiac failure. This is
because blood quickly enters the venous circulation from the arterial
circulation à combo of ­ preload back to right heart + poorer arterial - Answer = “Pulmonary arteriovenous fistula” (leading to high-output
perfusion distal to the fistula à compensatory ­ CO. failure); diagnosis is hereditary hemorrhagic telangiectasia. USMLE will
- AV fistulae can sometimes present with a continuous machinery murmur basically always show you a pic of red dots on the tongue/mouth or finger
similar to a PDA, since blood is continuously flowing through it. They in a patient with nosebleeds.
might say a continuous machinery-like murmur is auscultated in the leg at - Likewise, be aware intraosseous AV fistulae can occur in Paget, as
site of prior stab wound. mentioned before.
- As discussed earlier, they can present with bounding pulses similar to AR.
- Student says, “Well how am I supposed to know if it’s AV fistula then if it
sounds like other conditions too?” à by paying attention to HY points like,
“Is there lone S3 or S3/4 combo or EF >70%? Is there Hx of penetrating HY Murmur / ECG points for USMLE
trauma? Or does the patient have Paget? Etc.” - Described as “irregularly irregular” rhythm with absent p-waves.
- 2CK NBME Q shows you obscure angiogram of a fistula in the leg + tells
you there’s a continuous machinery murmur; they ask what most likely
determines prognosis in this patient à answer = “size of lesion.”

Arteriovenous fistula

- Notice how the QRS complexes are at random and irregular distances from
one another. This is the “irregularly irregular” pattern.
- AF is hugely important because it can cause turbulence/stasis within the left
atrium that leads to a LA mural thrombus formation. This thrombus can launch
off (i.e., become an embolus) and go to brain (stroke, TIA, retinal artery
Atrial fibrillation (AF) occlusion), SMA/IMA (acute mesenteric ischemia), and legs (acute limb
ischemia).
- AF HY in older patients, especially over 75. Vignette will usually be an older
patient with a stroke, TIA, or retinal artery occlusion, who has normal blood
pressure (this implies carotid stenosis is not the etiology for the embolus).
- AF usually is paroxysmal, which means it comes and goes. The vignette might
say the patient is 75 + had a TIA + BP normal + ECG shows sinus rhythm with no
abnormalities à next best step is Holter monitor (24-hour ambulatory ECG
monitor) to pick up the paroxysmal AF (e.g., when the patient goes home and
has dinner).
- NBME exam shows obscure image similar to above (without the arrow) + - After AF is diagnosed with regular ECG or Holter, 2CK wants echocardiography
they tell you there’s continuous murmur à answer = “size of lesion.” as the next best step to visualize the LA mural thrombus.
- Another NBME Q gives 45-year-old male will nosebleeds since - Patient who has severe abdominal pain in setting of AF or hyperthyroidism
adolescence + S3 heart sound + dyspnea + they show you pic of tongue; (which can cause AF), diagnosis is acute mesenteric ischemia; next best step is
they ask for the cause of dyspnea. mesenteric angiography; Tx is laparotomy if unstable (answer on NBME).

MEHLMANMEDICAL.COM 16 MEHLMANMEDICAL.COM 17
MEHLMANMEDICAL.COM MEHLMANMEDICAL.COM

- Severe pain in a leg + absent pulses in patient with irregularly irregular rhythm - VT is treated with anti-arrhythmics – i.e., amiodarone. If patient has coma or
= acute limb ischemia; USMLE wants “embolectomy” as answer. hemodynamic instability (low BP), the NBME answer is direct current
- Any structural abnormality of the heart, either due to LV hypertrophy, countershock or cardioversion (same thing).
ischemia, growth hormone/anabolic steroid use, prior MI, etc., can lead to AF. - Premature ventricular complex (PVC) is asked on 2CK.
- You need to know AF patient will get either aspirin or warfarin. This is
determined by the CHADS2 score. There are variations of the score, but the
simple CHADS2 suffices for USMLE à CHF, HTN, Age 75+, Diabetes,
Stroke/TIA/emboli. Each component is 1 point, but stroke/TIA/emboli is 2
points. If a patient has 0 or 1 points, give aspirin; if 2+ points, give warfarin.
- “Emboli” refers to Hx of AF leading to stroke, TIA, acute, mesenteric ischemia,
or acute limb ischemia – i.e., any Hx of embolic event. 2CK IM form gives short
vignette of 67F with chronic AF + Hx of acute limb ischemia + no other info
relating to CHADS, and answer is warfarin to prevent recurrence; aspirin is
wrong. - Note on the above strip, we have a wide complex (meaning ventricular in
- Some students will ask about NOACs, e.g., apixaban, etc., for non-valvular AF origin) that occurs earlier (hence premature). What they do on the NBME is
à I’ve never seen NBME care about this stuff. They seem to be pretty old- show you this strip and ask where this abnormality originates from, then the
school and just have warfarin as the answer, probably because there isn’t answer is just “ventricle.”
debate around whether it can be used; use of NOACs is less textbook. - Don’t treat PVCs on USMLE.
- AF patient should also be on rate control before rhythm control. The USMLE - Causes narrow / needle-shaped complexes. Make sure you’re able to contrast
actually doesn’t give a fuck about this component of management, although in this with VT above, which is wide-complex.
theory metoprolol or verapamil is standard. You could be aware for Step 3 that
flecainide is first-line for rhythm control if patients fail rate-control and have a
structurally normal heart and no coronary artery disease.
- NBME for 2CK has “electrical cardioversion” as the answer for patient with AF
who has hemodynamic instability (i.e., low BP). What you need to know is:
sometimes AF can trigger “rapid ventricular response,” where HR goes >150 and
low BP can occur.
- Has classic sawtooth appearance.
Supraventricular
tachycardia
(SVT)
- Notice the complexes are narrow / look like needles. This means the tachy
Atrial flutter originates above the ventricles (hence SVT).
- Treatment of SVT exceedingly HY on 2CK.
- First step is carotid massage (aka vagal maneuvers). In pediatrics, they can do
icepack to the face.
- Low yield for USMLE. I think it’s asked once on a 2CK NBME. But as student - If the above doesn’t work, the next step is give adenosine (not amiodarone).
you should know it exists / the basic ECG above. - Same as with VT, if the patient has coma or low BP, shocking the patient is the
- Causes wide-complex QRS complexes (>120 ms; normal is 80-120 ms). first step. In other words, for both SVT and VT, you must shock first in the
setting of coma or hemodynamic instability. It’s for stable SVT and VT that the
treatments differ on USMLE.
- Will present as ST-elevations in 3-4 contiguous leads.

Ventricular tachycardia
(VT)

Acute MI (STEMI)

- Exceedingly HY for 2CK that you know VT is wide-complex, whereas SVT is


narrow-complex. If you look at above ECG, even if you say, “No idea what I’m
looking at.” You can tell the complexes look wide like mountains in comparison
to a typical ECG.

MEHLMANMEDICAL.COM 18 MEHLMANMEDICAL.COM 19
MEHLMANMEDICAL.COM MEHLMANMEDICAL.COM

- The above is an inferior MI, as evidenced by ST-elevations in leads II, III, and - Patient with rheumatoid arthritis or SLE notably at risk for pericarditis. In other
aVF. The answer for the affected vessel is the posterior descending artery (PDA words, don’t get confused if they mention pericardial friction rub in vignette of
supplies the diaphragmatic surface of the heart); since >85% of people have RA or SLE; this is common.
right-dominant circulation (meaning the PDA comes of the right main coronary), - For cocaine use, they’ll say a 22-year-old male has chest pain after a night of
sometimes the answer for inferior MI can just be “right coronary artery.” heavy partying + ECG shows diffuse ST-elevations à Dx = pericarditis.
- If the Q says left-dominant circulation, the sequence USMLE wants is: left main - Uremic pericarditis is HY for 2CK. Q will give ultra-high creatinine and BUN and
coronary à left circumflex à PDA. say there’s a friction rub à treatment = hemodialysis.
- The apex of the heart is supplied by the left anterior descending artery (LAD). - Treatment for pericarditis is same as acute gout à NSAIDs, colchicine,
If there are ST-elevations in leads V1-V3, choose LAD as the answer. steroids.
- The left-lateral heart is supplied by the left circumflex artery. If there are ST- - Fibrinous pericarditis is post-MI and occurs as two types: 1) literally “post-MI
elevations in leads V4-V6 for lateral MI, choose left circumflex. fibrinous pericarditis,” which will simply be friction rub within days of an MI; 2)
- Reciprocal ST-depressions in the anterior leads V1-V3 can reflex posterior wall Dressler syndrome (antibody-mediated fibrinous pericarditis occurring 2-6
MI (i.e., we have “elevations” out the back of the heart, so they look like weeks post-MI).
depressions on the anterior wall leads). - ECG is first step in Dx of pericarditis, but USMLE wants echocardiography as
- As discussed earlier, if patient has MI followed by new-onset systolic murmur next best step in order to visualize a concomitant effusion that can occur
hours to days later, with or without dyspnea, that’s mitral regurg. sometimes. Vignette will give you stereotypical pericarditis + will ask for next
- Stroke-like presentation in patient who had MI weeks ago à “embolus from best step in diagnosis; ECG might not be listed and you’re like huh? à Answer is
ventricular septal aneurysm” (on 2CK form). echocardiography to look for potential effusion concomitant to the pericarditis.
- Most common cause of death due to MI is ventricular fibrillation (VF). - I should make note that chronic constrictive pericarditis is a separate condition
- Fibrosis of myocardium in the months-years post-MI increases risk of that doesn’t present with the standard pericarditis findings as described above.
arrhythmias such as AF, SVT, VT, etc. There’s no specific arrhythmia you need to - This is low-yield for USMLE, but students ask about it because it can be
memorize. Just know the risk is there in the future. confused with tamponade.
- Q waves on an ECG mean old MI / history of MI. The vignette might give you - There’s two ways this can show up:
patient who has light-headedness / fainting + they say patient has Q waves in II, 1) Tuberculosis is a classic cause; there may or may not be calcification
III, aVF, and the answer will be something like “paroxysmal supraventricular around the heart on imaging. So if you get a Q where patient has TB +
tachycardia.” Student thinks this specific arrhythmia matters, but it doesn’t. The some sort of heart-filling impairment à answer = chronic constrictive
point is that Hx of MI means patient is at risk for nearly any arrhythmia now. pericarditis.
- MI classically causes coagulative necrosis of the myocardium. 2) Kussmaul sign will be seen in the Q, where JVD occurs with
- With cardiogenic shock as a result of MI, the arrows USMLE wants are: ¯ inspiration rather than expiration.
cardiac output, ­ peripheral vascular resistance, ­ PCWP. - Normally, inspiration facilitates RA filling (¯ intrathoracic pressure à ­
- MI can lead to acute tubular necrosis from cardiogenic shock à acute drop in Chronic constrictive pulmonary vascular compliance/stretching à ­ high-low pressure gradient
renal perfusion. This is not pre-renal. I discuss this in detail in the renal section. pericarditis from right heart to the lungs à ¯ in afterload on RV from the lungs à blood
- First treatment for MI is aspirin. After aspirin is given, the next drug to give is moves easier from right heart to the lungs à blood is pulled easier from
clopidogrel (an ADP P2Y12 blocker) as dual anti-platelet therapy. SVC/IVC to the RA).
- USMLE wants you to know anyone with acute coronary syndrome (i.e., MI or - However, if there is ­ compressive force on the heart, the ­ in negative
unstable angina) gets coronary catheterization. This is answer on new 2CK intrathoracic pressure during inspiration is not transmitted to the right side of
NBME exam. the heart, so JVP does not ¯ (and can even paradoxically can ­).
- It’s to my observation that more extensive management of MI on USMLE, such - In tamponade, however, as discussed below, the ­ in negative intrathoracic
as use of beta-blockers, nitrates, morphine, oxygen, statin, percutaneous pressure during inspiration is able to be transmitted to the right side of the
coronary intervention, etc., isn’t assessed in detail. I can comment, however, heart, so Kussmaul sign does not occur. This is likely because in constrictive
that one 2CK Q wants you to know nitrates are contraindicated in right-heart pericarditis, the rigid pericardium prevents expansion of the right heart
MIs, which includes inferior MI in most people due to the right coronary altogether, whereas in tamponade, the pericardium isn’t rigid per se, but is just
supplying the PDA. This is because right-sided MIs are preload-dependent, filled with blood that can move/shift during the respiratory cycle, thereby
which means they need sufficient preload to maintain BP. allowing right heart expansion during inspiration.
- Percutaneous coronary intervention (PCI) is done in patients with STEMIs - Cardiac tamponade = pericardial effusion + low blood pressure.
within 90 minutes of reaching hospital. - What determines whether we have a tamponade or not is the rate of
- Shows up on ECG as diffuse ST-elevations (i.e., in all leads rather than 3-4 accumulation of the fluid, not the volume of the fluid – i.e., a stab wound or
contiguous leads as with MI). PR depressions can also be seen, but I’ve never post-MI LV free-wall rupture resulting in fast blood accumulation, even if
seen the USMLE give a fuck about the latter. smaller volume, might cause tamponade, but cancer resulting in slow, but large,
- Patient will have pain that’s worse when lying back, better when leaning Pericardial effusion / accumulation might not cause tamponade.
forward. In turn, the patient can present walking through the door bent over at Cardiac tamponade - Tamponade presents as Beck triad: 1) hypotension, 2) JVD, 3) muffled/distant
Pericarditis
the waist. heart sounds. The question will basically always give hypotension and JVD.
- Serous pericarditis will be post-viral, secondary to autoimmune disease, or due Occasionally they might not mention the heart sounds. But you need to
to cocaine use. memorize Beck triad as HY for tamponade.
- NBME Q gives pericarditis + a bunch of different organism types (i.e., - Pulsus paradoxus (i.e., drop in systolic BP >10 mm Hg with inspiration) is
bacterium, fungus, etc.), and answer is “virus.” classically associated with tamponade, although not frequently mentioned in

MEHLMANMEDICAL.COM 20 MEHLMANMEDICAL.COM 21
MEHLMANMEDICAL.COM MEHLMANMEDICAL.COM

vignettes. I’ve seen a 2CK NBME Q where they say “the pulsus paradoxus is <10 - Highest yield point is that if a patient has hyperkalemia and ECG changes, the
mm Hg,” which is their way of saying the Dx is not tamponade. I consider that Tx USMLE wants is IV calcium gluconate or calcium chloride, which stabilizes the
wording odd, but it’s what the vignette says. myocardium. Calcium gluconate is classic, but calcium chloride shows up as an
- ECG will show electrical alternans / low-voltage QRS complexes. answer on a 2CK NBME.
- Means hypokalemia.

U-wave

- Shows up on NBME 12 for 2CK in anorexia patient. First time I’ve ever seen it
show up anywhere on NBME material. But Q doesn’t ride on you knowing it
means hypokalemia to get it right. It’s HY and pass-level to know that purging
(anorexia or bulimia) causes hypokalemia anyway.
- You can see the amplitudes (i.e., heights) of the complexes are short. This - Seen in Wolff-Parkinson-White syndrome (WPW; accessory conduction
refers to “low-voltage.” You can also see the heights ever so slightly oscillate up pathway in heart that bypasses the AV node, resulting in reentrant SVT).
and down. This refers to electrical alternans. They show this ECG twice on 2CK - Classically described as a “slurred upstroke” of the QRS, where the PR interval
NBMEs. is shortened.
- If the Q asks for next best step in diagnosis, choose ECG as first step if listed If
not listed, then choose echocardiography, which confirms fluid over the heart.
- If the Q asks for next best step in management for tamponade when the
vignette is obvious, choose pericardiocentesis or pericardial window. USMLE
will not list both; it will be one or the other. NBME 8 offline for 2CK has
pericardial window as answer, where pericardiocentesis isn’t listed. Delta wave
- HY type of VT that has sinusoidal pattern on ECG.

- Both the delta-wave and WPW have basically nonexistent yieldness on USMLE,
Torsades de pointes but I mention them here so you are minimally aware.
(TdP) - USMLE wants you to know this can be caused by some anti-arrhythmic agents, - They mean hypothermia. You don’t need to be able to identify on ECG. Just
such as the sodium- and potassium-channel blockers, such as quinidine and J waves know they exist, as they show up in a 2CK vignette where patient has body
ibutilide, respectively. They ask this directly on the NBME exam, where Q will temperature of 89.6 F (not 98.6).
say patient is given ibutilide + what is he now at increased risk of à answer =
torsades.
- QT prolongation is risk factor for development of TdP. Agents such as anti-
psychotics, macrolides, and metoclopramide prolong the QT. HY Heart block points for USMLE
- Tx USMLE wants is magnesium (asked directly on new 2CK form), which - Prolonged PR interval (>200 ms). Should normally be 80-120 ms.
stabilizes the myocardium in TdP.
- Seen in hyperkalemia.

First degree
Peaked T wave

- Note that above on the ECG, the PR-segment in particular (just prior to
the QRS complex) is extra-long.
- Asked once on one of the 2CK forms, where they show the ECG. - Not really assessed on USMLE. Just know the definition.
- Don’t treat on USMLE.

MEHLMANMEDICAL.COM 22 MEHLMANMEDICAL.COM 23
MEHLMANMEDICAL.COM MEHLMANMEDICAL.COM

- Gradually prolonging PR interval until QRS drops. Then cycle repeats. HY Cardiomyopathy points for USMLE
- Can be isolated ventricular or diffuse 4-chamber dilation. Causes are
multifarious, but a key feature is systolic dysfunction, where ejection
Second degree Mobitz type I fraction is reduced (i.e., <55%, where normal range is 55-70).
(aka Wenckebach) - CXR shows enlarged cardiac silhouette.

- Don’t treat on USMLE.


- No gradual prolongation of PR interval, followed by a random dropping
of the QRS.

- Can also sometimes occur as patterns of 2:1, 3:1, etc., where there will
be a P to QRS ratio of 2:1 or 3:1, etc.
Second degree Mobitz type II Dilated (DCM)

- An S3 heart sound can sometimes be heard.


- Cardiac exam shows lateralized apex beat. Can be described as the
point of maximal impulse being in the anterior axillary line. Should be
- Regardless as to whether the dropped QRS is random or in a numerical noted that this lateralization just means an enlarged LV, so it is non-
pattern, there is no gradual prolongation of the QRS before the dropped specific, and can also be seen in LV hypertrophy from any cause. But
complex. many vignettes will mention it.
- More dangerous than Mobitz I. This is because Mobitz II has higher - The arrows USMLE wants are: ¯ EF; ­ LVEDV; ­ LVEDP. I discuss this
chance of progression into type III heart block. stuff in more detail in my HY Arrows PDF.
- Treatment on USMLE is insertion of pacemaker. This is asked on a new - Causes of DCM are ABCD:
2CK NBME exam. - A: Alcohol.
- Two things you want to look for on ECG: - B: Wet Beriberi (thiamine deficiency).
- 1) Ultra-slow HR (i.e., 30-40). You’ll see the QRS’s are super far apart. - C: Coxsackie B virus, Cocaine, Chagas disease.
This is the ventricular escape rhythm. - D: Drugs à doxorubicin (aka Adriamycin).
- 2) No relationship between the P-waves and QRS complexes. - Other notable causes are:
- Pregnancy (peripartum cardiomyopathy); hemochromatosis; rheumatic
heart disease (myocarditis leading to DCM).
- Heart failure due to systemic hypertension.
Third degree - Characterized by diastolic dysfunction (the heart can pump just fine but
cannot expand as easily).
- Can be associated with S4 heart sound.
- The arrows USMLE wants are: « EF; « LVEDV; ­ LVEDP.
- Treatment on USMLE is insertion of pacemaker. - Ejection fraction is normal because the heart can pump perfectly fine.
- So what you want to remember is that Mobitz II and 3rd-degree are the - Students get confused about LVEDV, thinking it should be low, if the
heart cannot expand as easily. But this is not the case for USMLE. They
ones where we insert pacemaker; 1st-degree and Mobitz I we don’t.
want you to know normal volume can be achieved; it just merely requires
Hypertrophic (HCM) more force/pressure to get there.
- As with DCM, the apex beat / point of maximal impulse can be
lateralized, which merely reflects LV hypertrophy.
- As described earlier, paradoxical splitting of S2, left-axis deviation on
ECG, and LBBB can all be seen due to LVH. When you see these findings
in vignettes, don’t get confused. They just mean LVH.
- Often associated with hypertensive retinopathy (fundoscopy shows
narrowing of retinal vessels, flame hemorrhages, and “AV-nicking”),
hypertensive nephropathy (hyperplastic arteriolosclerosis with increased
creatinine).

MEHLMANMEDICAL.COM 24 MEHLMANMEDICAL.COM 25
MEHLMANMEDICAL.COM MEHLMANMEDICAL.COM

- Caused by mutations in b-myosin heavy-chain gene; autosomal - Statins have 2 HY MOAs on USMLE: 1) inhibit HMG-CoA reductase; 2) upregulate LDL receptors on
dominant; results in disordered/disarrayed myocardial fibers. hepatocytes.
- HOCM causes asymmetric septal hypertrophy that results in the - Ezetimibe blocks cholesterol absorption in the small bowel.
anterior mitral valve leaflet obstructing the LV outflow tract (so it can - Bile acid sequestrants (e.g., cholestyramine) result in the liver pulling more cholesterol out of the blood.
sound similar to aortic stenosis). - Fibrates upregulate PPAR-a and lipoprotein lipase; best drugs to decrease triglycerides.
- Classically sudden death in young athlete; cause of death is ventricular
fibrillation due to acute left heart strain in the setting of fast heart rate.
- Presents with systolic murmur that worsens with Valsalva or standing
(aortic stenosis, in contrast, gets softer or experiences no change with HY Angina points for IM
Valsalva) or standing. - Chest pain that occurs predictably with exercise.
- HOCM and MVP are the only two murmurs that get worse with less - Due to atherosclerotic plaques causing >70% occlusion; can be calcific.
volume in the heart. All other murmurs get worse with more volume. - Classically causes ST depressions on ECG.
Hypertrophic obstructive
Valsalva increases intra-thoracic pressure and decreases venous return, - Nitrates (e.g., sublingual isosorbide dinitrate) used as Tx à nitrates “donate”
(HOCM)
so there’s less volume in the heart. Standing simply decreases venous nitric oxide (NO) that upregulates guanylyl cyclase within venous smooth
return. muscle à increased cGMP à relaxation of venous smooth muscle à increased
- NBME wants you to know that ­ HR from exercise or stress means Stable angina
venous pooling of blood à decreased venous return à decreased myocardial
“diastole is shortened more than systole.” oxygen demand à mitigation of chest pain.
- Beta-blockers (metoprolol or propranolol are both answers on NBME) - Nitrates are contraindicated with PDE-5 inhibitors (e.g., Viagra) due to risk of
are given to slow heart rate, which maximizes diastolic filling and low blood pressure.
decreasing symptoms / risk of death. - Sodium nitroprusside used for hypertensive emergencies dilates arterioles in
- Implantable cardioverter-defibrillator (ICD) is indicated if patient addition to the veins. If USMLE asks you where this drug acts, choose arterioles.
develops any type of arrythmia, syncope, low blood pressure, or LV wall - Chest pain that is unpredictable and can occur at rest.
thickness >30mm. - Due to partial rupture of atherosclerotic plaque leading to partial occlusion.
- Septal myectomy is done if the left ventricular outflow tract pressure Unstable angina - ST depressions on ECG.
gradient is >50mm Hg. But it should be noted ICD is usually first-line. - Diltiazem is answer on new 2CK NBME for patient with unstable angina.
- Heart failure due to diastolic dysfunction, where HTN is not the cause. - Patients need cardiac catheterization.
- JVD is HY for RCM. An S4 can also be seen. The heart will not be dilated. - Vasospastic angina that occurs at rest (i.e., watching TV or while sleeping) in
- HY causes are Hx of radiation (leads to fibrosis), amyloidosis, and younger adults; it is not caused by atherosclerosis.
hemochromatosis. - ST elevations are seen on ECG.
- Student might say, “I thought you said hemochromatosis was DCM. So - You must know that Prinzmetal is also known as variant angina pectoris. There
if we have to choose on the exam, which one is it?” The answer is, Prinzmetal angina is an NBME Q that gives vignette of Prinzmetal, but answer is “variant angina
whichever the vignette gives you. If they say a large cardiac silhouette (variant angina pectoris) pectoris.”
Restrictive (RCM)
with an S3 and lateralized apex beat, that’s DCM. If they say JVD + S4 + - Treatment is nitrates (can cause coronary artery dilation unrelated to the
nothing about a lateralized apex beat, you know it’s RCM. venous pooling effects) or dihydropyridine calcium channel blockers (e.g.,
- Amyloidosis is protein depositing where it shouldn’t be depositing. nifedipine). Avoid a1-agonists in these patients (cause vasoconstriction), as well
Highest yield cause of amyloidosis on USMLE is multiple myeloma, which as non-selective b-blockers like propranolol (can cause unopposed a effects).
will lead to RCM.
- Since RCM is diastolic dysfunction, the arrows are the same as HCM,
which are: « EF; « LVEDV; ­ LVEDP.
Hypertensive Emergency + urgency
- HTN >180/120 + signs of end-organ damage.
- The latter can be hypertensive encephalopathy (confusion), nephropathy (poor renal
Atherosclerosis basic points for IM function tests), retinopathy, acute heart failure, etc.
- Most acceleratory risk factors are diabetes mellitus (I and II), followed by smoking, followed by HTN, in Emergency - BP should be ¯ by no more than 20-25% in the first hour, as drastic ¯ can compromise
that order. perfusion to the brain and vital organs.
- HTN is most common risk factor, but DM and smoking are worse. I talk a lot about this stuff in my HY Risk - Blood pressure should be brought under 160/100 by 24-48 hours.
Factors PDF if you want extensive detail. - Drugs used are IV sodium nitroprusside, IV nicardipine, IV labetalol, and oral captopril.
- HTN is most acceleratory specifically for carotid stenosis (systolic impulse pounds carotids à endothelial - HTN >180/120 + no signs of end-organ damage.
damage). Urgency - Blood pressure should be brought under 160/100 by 24-48 hours.
- Stroke, TIA, or retinal artery occlusion in patient with high BP is due to carotid plaque launching off to the - Drugs used are IV sodium nitroprusside, IV nicardipine, IV labetalol, and oral captopril.
brain/eye. If patient has normal BP, think AF instead, with left atrial mural thrombus launching off.
- Patient over 50 with Hx of cardiovascular risk factors who now has accelerated HTN, think renal artery
stenosis (narrowing due to atherosclerosis).
- Plaques can calcify. The more calcium there is in a plaque, the more mature it is often considered to be.
Calcium scoring is routinely done in patients who have coronary artery disease in the assessment of plaque
progression.

MEHLMANMEDICAL.COM 26 MEHLMANMEDICAL.COM 27
MEHLMANMEDICAL.COM MEHLMANMEDICAL.COM

Shock types 4) Hx of heart transplant with valvular regurgitation of any kind.


Systemic vascular Pulmonary capillary wedge pressure - Highest yield point for USMLE about endocarditis prophylaxis is that mitral valve
Cardiac output (CO)
resistance (SVR) (PCWP) prolapse (MVP) and valve regurgitations or stenoses are not an indication. In
Cardiogenic ¯ ­ ­ other words, do not give prophylaxis if the patient has MVP, MR, AS, etc. In
Hypovolemic ¯ ­ ¯ addition, bicuspid aortic valve is not an indication.
Septic ­ (early) / ¯ (late) ¯ ¯
Anaphylactic ­ (early) / ¯ (late) ¯ ¯
Neurogenic ¯ ¯ ¯
Obstructive ¯ ­ ¯ Rheumatic heart disease (rheumatic fever) HY points
- Septic, anaphylactic, and neurogenic are all under the envelope of distributive shock. - Strep pyogenes (Group A Strep) oropharyngeal infection results in production of antibodies against S.
- ­ PCWP for cardiogenic is one of the highest yield path points on USMLE. pyogenes’ M-protein that cross-react with the mitral valve (i.e., molecular mimicry; type II hypersensitivity).
- For deeper explanations, go to my HY Arrows PDF. - Can occur with the aortic valve in theory, but on USMLE, it is always mitral valve.
- Results in mitral regurgitation acutely and mitral stenosis late, as discussed earlier.
- Presents as JONES (J©NES) à Joints (polyarthritis), © Carditis, subcutaneous Nodules, Erythema
marginatum (annular, serpent-like rash), Sydenham chorea (autoimmune basal ganglia dysfunction that
HY Endocarditis points results in dance-like movements of the limbs).
- Cutaneous Group A Strep infections don’t cause rheumatic fever, but can still cause PSGN.
- Bacterial infection of valve in patient with no previous heart valve problem.
- Treatment is penicillin.
- Caused by Staph aureus on USMLE.
- Left-sided valves (i.e., aortic and mitral) most commonly affected because of
greater pressure changes (i.e., from high to low) within left heart, resulting in
Acute endocarditis
turbulence that enables seeding.
- IV drug users à venous blood inoculated with S. aureus à travels to heart and Conditions confused for cardiac path
causes vegetation of tricuspid valve. - NBME loves trying to make you think this is an MI.
- Staph aureus is coagulase positive. - They’ll give you young, healthy patient who feels doom / like he or she is
- Bacterial infection of valve in patient with history of valve abnormality (i.e., going to die.
congenital bicuspid aortic valve, Hx of rheumatic heart disease). - Sometimes they mention in stem Hx of MI in family as distraction.
- Caused by Strep viridans on USMLE. You need to know S. viridans is can be Panic attack - They can say patient has mid-systolic click, as discussed earlier, and then
Subacute endocarditis they ask for cause of patient’s symptoms à answer = panic disorder, not
further broken down into: S. sanguinis, S. mutans, and S. mitis.
- Hx of dental procedure is HY precipitating event, where inoculation of blood MVP. Student gets confused, but MVP is almost always asymptomatic, where
occurs via oral cavity à previously abnormal valve gets seeded. panic attack is clearly cause of the patient effusively hyperventilating.
- New-onset murmur + fever = endocarditis till proven otherwise on USMLE. - Treat with benzo.
- Reactive thrombocytosis (i.e., high platelets) can occur due to infection. This is - Orthostatic hypotension is defined as intravascular fluid depletion causing a
not unique to endocarditis, but it is to my observation USMLE likes endocarditis drop of systolic BP >20 mmHg and diastolic BP >10 mmHg when going from
as a notable etiology for it. In other words, if you get an endocarditis question supine to standing.
and you’re like, “Why the fuck are platelets 900,000?” (NR 150-450,000), don’t be Orthostasis - Shows up on 2CK IM form as exactly a drop of 20 and 10, respectively, for
confused. systolic and diastolic BPs in a patient with fainting à answer = “intravascular
Random points fluid depletion.”
- Hematuria can occur from vegetations that launch off to the kidney.
- Endocarditis + stroke-like episode (i.e., focal neurologic signs) = septic embolus, - Diuretic use is big risk factor.
where a vegetation has launched off to the brain. - Fainting in response to stressor (e.g., emotional trigger).
- Janeway lesions, Osler nodes, splinter hemorrhages, etc., are low-yield for - Stress triggers an initial sympathetic response, which in turn triggers a
USMLE and mainly just school of medicine talking points. compensatory parasympathetic response. This latter response is excessive in
- HACEK organisms nonexistent on USMLE. Vasovagal syncope some people, where the peripheral arterioles dilate and the heart slows too
- Blood cultures before antibiotics is important for 2CK. much à decreased cerebral perfusion à lightheadedness/fainting.
- Transesophageal echocardiography (TEE) confirms diagnosis after blood - 2CK wants you to know a tilt-table test can be used to diagnose, where a
cultures. Transthoracic echocardiography (TTE) is not done for endocarditis. reproduction of symptoms can occur.
- For 2CK, empiric treatment for endocarditis is vancomycin, PLUS either - USMLE likes this for both Steps 1 and 2.
gentamicin or ampicillin/sulbactam. - They’ll say dude was shaving then got lightheadedness or fainted.
Carotid sinus
- Vancomycin targets gram-positives (including MRSA). Gentamicin targets gram- Mechanism is ­ stretch of carotid sinus baroreceptors à ­ afferent CN IX
hypersensitivity
negatives. firing to solitary nucleus of the medulla à ­ efferent CN X parasympathetic
Management firing down to cardiac nodal tissue à ¯ HR à ¯ CO à ¯ cerebral perfusion.
- Endocarditis prophylaxis given prior to a dental procedure is usually ampicillin or
a second-generation cephalosporin, such as cefoxitin. - Inflammation of cartilage at rib joints.
- Indications for endocarditis prophylaxis are: - Will present as chest pain that worsens with palpation or when patient
1) Hx of endocarditis (obvious); Costochondritis reaches over the head or behind the back. These two findings are clear
2) If there is any prosthetic material in the heart whatsoever; indicators we have an MSK condition, not cardiac.
3) If there is any congenital cyanotic heart disease that has not been completely - Can be idiopathic, caused by strain (e.g., at the gym), or even post-viral.
repaired (if it’s been completely repaired with prosthetics, give prophylaxis);

MEHLMANMEDICAL.COM 28 MEHLMANMEDICAL.COM 29
MEHLMANMEDICAL.COM MEHLMANMEDICAL.COM

- MSK condition asked twice on 2CK material that has nothing to do with the - Congestion of venous system usually from valvular incompetence; idiopathic /
lungs, despite the name. familial; varicose veins are one type of venous disease and are not synonymous;
- This is viral infection (Coxsackie B) causing sharp lateral chest pain due to patients can have venous disease without varicosities.
Pleurodynia
intercostal muscle spasm. Sometimes students choose pericarditis, etc., even - Peripheral pulses are normal (those reflect arteries, not veins).
though the presentations are completely disparate. - Lower legs demonstrate “brawny edema,” which is a brown, hemosiderin-
- Creatine kinase can be elevated in stem due to ­ tone of muscle. laden edema due to ­ pressure / micro-extravasations; hyperpigmentatory
- Viral infection causing inflammation of the pleura (layers covering the lungs), changes resulting in brown/red skin is known as stasis dermatitis, aka post-
leading to sharp chest pain. phlebitic syndrome; the latter is a term is asked on 2CK, so know the annoying
Viral pleurisy
- If this is the answer, CK will be normal (unlike pleurodynia, because it’s not vocab.
MSK). - Venous ulcers are large and sloughy, and located at the malleoli.
- Can cause angina-like pain in patient without cardiovascular disease.
Diffuse esophageal spasm
- I discuss this in detail in the Gastro section.
- Can present as chest pain confused for MI. ECG will be normal, clearly.
Gastroesophageal reflux
- I discuss GERD in detail in the Gastro section.

Arterial vs venous disease


- Caused by atherosclerotic disease; presents as diminished peripheral pulses in
patient over 50 who has risk factors, e.g., diabetes, smoking, HTN. Venous disease
- Lower legs can be shiny and glabrous (trophic changes).
- Arterial ulcers are small and punched-out; located on tops/bottoms of feet and
toes.

- Diagnose with venous duplex ultrasonography of the legs; first treatment is


compression stockings. Never choose answers such as venous stripping or glue
agents, etc.
- Venous disease ­ risk for DVT and superficial thrombophlebitis. If patient has
active DVT or STP, answer = subcutaneous enoxaparin (heparin) over
compression stockings.
- Surgery is indicated if there are significant skin changes, venous ulcers, or for
symptomatic varicose veins (i.e., pain, itching, swelling, cramps).
- You could be aware that sclerotherapy is often chosen as the first surgery,
where a sclerosing (scarring) agent is injected into the vein, causing it to close
Arterial disease where the blood is re-routed to other veins. But basically always, just
compression stockings, or rarely heparin for DVT or STP, is the answer.
- Ankle-brachial indices (ABIs) are first step in diagnosis (exceedingly HY on 2CK),
which compare BP in ankle to the arm; if <0.9, this reflects ¯ peripheral blood
flow due to atherosclerosis.
Cardiac stress test points
- If ABIs are not listed as first step in diagnosis for whatever reason, choose
- Most 2CK Qs that ask about stress tests are in the context of evaluating patients for perioperative MI risk.
Doppler ultrasound. There is one 2CK NBME Q where this is the case.
- It is rare the Q will force you to choose between different types of stress tests. 4/5 Qs will just list one
- After ABIs, next step is exercise stress test (if listed) in order to determine
stress test, where it is simply assessing, “Do you know a stress test should be done, period, in this scenario.”
exercise tolerance. If not listed, go straight to “recommend an exercise /
- Stress tests are also done for peripheral arterial disease prior to recommending an exercise/walking
walking program.” Do not choose cilostazol first or arteriography as answers.
program (as mentioned above).
- One NBME form has “prescription for an exercise program” as the answer.
- Most common stress test.
Students say, “Why does it say ‘prescription’?” No fucking idea. Ok?
- The answer on USMLE for patients who have stable angina, where you’re looking for ST
- Surgery is indicated in the event of critical limb ischemia, which is when there
depressions (i.e., evidence of ischemia) with exertion.
is chronic ischemic rest pain, ulcers, or gangrene.
Exercise ECG - Requires a patient has a normal baseline ECG in order to perform.
- First surgical intervention is usually angioplasty +/- stenting. Endarterectomy
- In other words, the Q will give you a big 15-line paragraph + mention in the last line that
and bypass surgery are indicated for more severe blockages.
the patient’s baseline ECG shows, e.g., a LBBB from a year ago that’s unchanged. This
- All patients with arterial disease should be on triad of 1) ACEi/ARB, 2) statin, 3)
means ECG stress test is wrong in this situation, since you need to have a normal ECG to
anti-platelet therapy (same as carotid stenosis). This is unrelated to the
management sequence of exercise program à cilostazol à surgery.

MEHLMANMEDICAL.COM 30 MEHLMANMEDICAL.COM 31
MEHLMANMEDICAL.COM MEHLMANMEDICAL.COM

do it. The 1/5 Qs that force you to choose between stress tests want you to know this HY Vasculitides (fancy word that’s pleural for vasculitis)
detail, basically, where you just choose the non-ECG stress test instead. - Formerly known as Wegener granulomatosis.
- Used to look for heart failure (i.e., ¯ EF) with exertion, not overt ischemia. - Answer on USMLE for adult with triad of 1) hematuria, 2) hemoptysis, and 3)
- In other words, the answer on USMLE for patients who don’t get chest pain with exertion “head-itis” – i.e., any problem with the head, such as nasal septal perforation,
Granulomatosis with
Exercise echo (i.e., don’t have stable angina), but who get shortness of breath with exertion. This mastoiditis, sinusitis, otitis.
polyangiitis
reflects, at a minimum, left heart decompensation with possible ¯ EF. - Associated with cANCA and anti-proteinase 3 (anti-PR3) antibodies.
- Also the answer for patients who have abnormal baseline ECG. - Causes “necrotizing glomerulonephritis” that can lead to rapidly progressive
- Refers to numerous answer choices on USMLE – i.e., dobutamine-echo, dipyridamole- glomerulonephritis (RPGN).
thallium. - Formerly known as Churg-Strauss.
- The answer on USMLE for patients who cannot exercise, such as in the setting of angina Eosinophilic - Presents as combo of asthma + eosinophilia +/- head-itis.
when merely walking up a single flight of steps, or in patients imminently undergoing granulomatosis with - Head-itis always seen in Wegener vignettes, but maybe only ~50% of CS Qs.
major surgery (e.g., AAA repair), where perioperative MI risk needs to be assessed. I have polyangiitis - Renal involvement rare for CS.
seen both of these scenarios on 2CK forms. - Associated with pANCA and anti-myeloperoxidase (anti-MPO) antibodies.
- The USMLE will typically not force you to choose between stress tests. As I mentioned at - Will just present as hematuria in a patient who is pANCA / anti-MPO (+).
Pharmacologic Microscopic polyangiitis
the top of this table, they will usually just have the pharmacologic stress test as the only - Similar to Wegener, can cause RPGN.
one listed. - The above three conditions can be associated with a weird neuropathy called mononeuritis multiplex,
- Dobutamine is a b1-agonist that stimulates the heart (i.e., ­ oxygen demand). Echo can which means neuropathy of “one large nerve in many locations” – e.g., wrist drop + foot drop in same
then be done to look for ¯ EF (i.e., heart failure). patient.
- Dipyridamole is a phosphodiesterase inhibitor that dilates arterioles. HR goes up to - You don’t need to worry about the fancy term “mononeuritis multiplex,” but what I do want you to be
compensate, thereby ­ myocardial oxygen demand. Thallium is then used to look at aware of is that neuropathy will sometimes show up in these vasculitis vignettes, so don’t be confused
perfusion of the myocardium. about it.
- “Cardiac scintigraphy” is a broad term that refers to any evaluation of the heart in which - Medium-vessel vasculitis that causes a “string of pearls” appearance of the
some form of radiotracer is used (i.e., thallium, technetium, sestamibi). renal vessels.
- This is the same as pharmacologic stress test for all intents and purposes on USMLE, even
Cardiac
though technically it need not require myocardium is stimulated and can just be used to
scintigraphy
look at blood flow to the heart in the resting state.
- The point is: This is an answer on 2CK sometimes as just another way of them writing
“pharmacologic stress test.” Choose it if the patient cannot exercise.
- “Myocardial perfusion scan” is one type of cardiac scintigraphy that evaluates blood flow
to myocardium. It is non-invasive, whereas coronary angiography is invasive and evaluates
Myocardial
coronary blood flow via the use of a catheter.
perfusion scan
- This is interchangeable with cardiac scintigraphy and pharmacologic stress test on USMLE
for all intents and purposes.
Polyarteritis nodosa (PAN)

Knee trauma causing popliteal arterial injury


- If Q gives you MVA where the knee is injured + absent pulses distally, the sequence of
answers they want is knee relocation first, followed by arteriography to look for
popliteal artery injury.
Knee dislocation
- In one of the 2CK Qs, they already tell you the knee is relocated, then the answer is
“arteriography with runoff.” Students say, “what’s the runoff part?” No fucking idea,
it’s just what they want. - Causes fibrinoid necrosis, which means it looks like fibrin but it ain’t fibrin.
- If Q gives gunshot wound to the knee + absent distal pulses, go straight to “surgical - One NBME Q has “segmental ischemic necrosis” as the answer.
Penetrating exploration” as the answer. - Can be caused by hepatitis B.
trauma - This could be thought of as the knee-equivalent of a gunshot wound to the abdomen, - For whatever reason, USMLE wants you to know PAN spares the lungs – i.e.,
where straight to laparotomy (even if patient is stable) is the answer. it does not affect the pulmonary vessels.
- Aka “pulseless disease.” Classically affects Asian women 40s or younger.
- Inflammation of large vessels, including the aorta.
Takayasu arteritis
- Always affects the subclavian arteries (which supply the arms), which is why
it can cause weakly, or non-palpable, pulse in the upper extremities.
- Aka giant cell arteritis.
- 9/10 Qs will be painful unilateral headache in patient over 50. I’ve seen one
Temporal arteritis
Q on NBME where it’s bilateral.
- Flares can be associated with low-grade fever and high ESR.

MEHLMANMEDICAL.COM 32 MEHLMANMEDICAL.COM 33
MEHLMANMEDICAL.COM MEHLMANMEDICAL.COM

- Patients can get proximal muscle pain and stiffness. This is polymyalgia - There are two 2CK Qs on this stuff. One just mentions a guy going into
rheumatica (PMR). The two do not always go together, but the association is surgery who has Hx of venous stasis à answer = “prophylactic heparin dose”;
HY. (Do not confuse PMR with polymyositis. The latter will present with ­ CK “therapeutic heparin dose” is wrong answer.
and/or proximal muscle weakness on physical exam. PMR won’t have either of - The second question gives a guy who’s already on prophylactic heparin but
these findings. I talk about this stuff in detail my MSK notes.) gets a DVT anyway. The answer is then “heparin.” It’s weird because students
- Patients can get pain with chewing. This is jaw claudication (pain with are like, “Wait what? He’s already on heparin though.” And I’m like, yeah, but
chewing). what they mean is, we have to give therapeutic dose now for the active DVT,
- Highest yield point is we give steroids before biopsy in order to prevent which is higher dose.
blindness. - DVT can rarely cause stroke if an ASD is present (paradoxical embolus).
- An NBME has “ischemic optic neuropathy” as the answer for what Dumb and low-yield, but it shows up, and students get fanatical over it.
complication we’re trying to prevent by giving steroids in temporal arteritis. - Thrombophlebitis means inflammation of a vein.
- IV methylprednisolone is typically the steroid given, since it’s faster than oral - Post-surgery, this is usually due to changes in hemostasis and coagulability.
Post-op migratory
prednisone. - Will present as pink/red painful lesions appearing asymmetrically on the
- It’s to my observation many 2CK NBME Qs will give the answer as something limbs within days of surgery. You just need to be able to diagnose this.
like, “Steroids now and then biopsy within 3 days,” or “IV methylprednisolone - Migratory thrombophlebitis classically due to head of pancreas
Trousseau sign of
and biopsy within a week.” Students ask about the time frames, but for adenocarcinoma. But this can also be seen with adenocarcinomas in general,
malignancy
whatever reason USMLE will give scattered/varied answers like that. e.g., pulmonary.
- Another 2CK Q gives easy vignette of temporal arteritis and then asks next - Shows up on 2CK form as patient who had a catheter in and then develops a
best step in diagnosis à answer = biopsy. Steroids aren’t part of the answer. Catheter-associated septic 4-cm indurated, painful, fluctuant cord in his arm (refers to vein).
Makes sense, since they’re asking for a diagnostic step. thrombophlebitis (CAST) - Answer = “excision of vein.” Obscure question, but not my opinion. Take it
- Aka Buerger disease; technically a vasculitis. up with NBME if you think it’s weird.
- Dry gangrene of the fingers or toes seen generally in male over 30 who’s a The answer on USMLE in a woman who has post-partum endometritis (fever +
Thromboangiitis obliterans heavy smoker. tender lower abdomen) with persistent fever >48 hours despite antibiotics.”
- Treatment is smoking cessation. - Endometritis can lead to ­ risk of local infective clots in the ovarian veins.
- Don’t confuse with Berger disease, which is IgA nephropathy. Pelvic septic
- If they give you a post-partum woman with sepsis (i.e., SIRS + infection), but
thrombophlebitis (PST)
- Tertiary syphilis can cause ascending aortitis + aortic aneurysm. the vignette doesn’t fit PST as described above, the answer is “puerperal
Ascending aortitis
- Causes “tree-barking” of the aorta. sepsis” on the 2CK form. The latter is a more general term and can refer to
many causes of post-partum sepsis (including PST confusingly enough).
- Painful palpable cord in the ankle that may or may not track up to the knee.
- Seen in patients with venous insufficiency.
Thrombophlebitis - Answer is “subcutaneous enoxaparin.” Compression stockings are typically
Superficial
- DVT will be unilateral thigh or lower leg swelling in patient with risk factors the answer for first step in venous insufficiency, but if you have an active ST or
thrombophlebitis
such as: post-surgery, prolonged sedentation, OCP use, Hx of thrombotic DVT, heparin must be given as first step.
disorders (e.g., Factor V Leiden, prothrombin mutation). - There will occasionally be some intentional redundancy on my end with
- Virchow triad for ­ DVT risk: 1) venous stasis (e.g., post-surgery sedentation), things I write in this doc if I believe they’re HY enough (as with this).
2) hypercoagulable state (e.g., estrogen use, underlying malignancy), 3)
endothelial damage (i.e., smoking).
- OCPs contraindicated in smokers over 35 because estrogen causes
hypercoagulable state for two reasons: 1) estrogen upregulates fibrinogen; 2) HY familial dyslipidemias for IM
estrogen upregulates factors Va and VIIIa. Condition Mechanism HY point(s)
- USMLE loves nephrotic syndrome as cause of DVT (loss of antithrombin III in - ­ chylomicrons + TGAs.
the urine à hypercoagulable state). Hyperchylomicronemia Deficiency of lipoprotein lipase - Pancreatitis (abdo, not chest, pain).
- Antiphospholipid syndrome à DVTs despite paradoxical ­ PTT (i.e., if PTT is (AR) or Apolipoprotein C-II - Xanthomas.
Deep vein thrombosis
high, you’d think you have bleeding diathesis, not thromboses); may or may - Plasma appears “creamy”.
not be due to SLE. Antibodies against phospholipids cause in vivo clumping of - ­ LDL.
platelets + ­ clot initiation, but disruption of in vitro PTT assay means ­ PTT. - LDL usually 3-400 in heterozygotes, with
- Major danger is DVT can embolize to lungs causing PE à acute-onset MI/death in 30-40s (answer = deficiency of
shortness of breath and tachycardia + death if saddle embolus. Deficiency (heterozygous) or
Hypercholesterolemia functional LDL receptor).
- Homan sign can mean DVT, which is pain in the calf with dorsiflexion of foot. absence (homozygous) of LDL
(AD) - LDL usually 700-1000 in homozygotes, with
- Diagnose DVT with duplex venous ultrasound of the leg/calf. receptor or Apo B-100
MI in teens (answer = absence of functional
- Treatment is heparin. LDL receptor).
- Harder stuff for 2CK is that they care about prophylactic vs therapeutic doses - Xanthomas.
of heparin. Prophylactic dose is lower-dose and is used perioperatively in Hypertriglyceridemia - ­ TGAs.
patients with venous disease/stasis or who are high risk. If a patient has an ­ Hepatic production of VLDL
(AD) - Pancreatitis.
actual full-blown DVT, however, give therapeutic dose, which is higher-dose.

MEHLMANMEDICAL.COM 34 MEHLMANMEDICAL.COM 35
MEHLMANMEDICAL.COM MEHLMANMEDICAL.COM

Random HY cardio pharm points - Hypokalemia can cause toxicity. This is because digoxin binds to extracellular K+ binding
- Non-dihydropyridine calcium channel blocker (acts on nodal calcium channels). site, so if less K+ is around to compete, lower dose is needed to induce effect.
- Causes constipation. - Toxicity presents classically as yellow/wavy “Vincent van Gogh” vision.
Verapamil
- Used for AF for rate control sometimes in place of metoprolol (don’t worry about the - Does not decrease mortality in heart failure.
use-case; USMLE doesn’t give a fuck; just know MOA and side-effect). - Classic loop diuretic that inhibits the apical Na+/K+/2Cl- ATPase symporter on the thick
- Dihydropyridine calcium channel blocker (acts on arteriolar calcium channels à dilates ascending limb. Apical means side of urine. Basolateral means side of blood.
arterioles à ¯ peripheral vascular resistance à ¯ BP). - Most efficacious diuretic at enabling fluid unloading for pulmonary and peripheral
- Causes peripheral edema / fluid retention (HY on Family Med). edema. Do not decrease mortality in cardiac patients.
Nifedipine
- Used for essential HTN in patients without diabetes, atherosclerotic disease, or renal - Answer on USMLE for patient with heart failure who has ¯ O2 sats who “refuses to lie
disease (if that sounds confusing, I talk about this in extensive detail in my HY Risk supine/back on the gurney” because he/she can’t breathe. Q might say CXR or exam
Factors PDF). shows fluid 2/3 up the lung fields.
- Beta-1 is on myocardium. - Application of loops can be confused by students with ACEi/ARB. For instance, the
- Beta-2 is on peripheral arterioles. latter are used first-line to ­ EF in heart failure even if O2 sats are low (¯ AT-II
- There are other binding spots, but the above are what USMLE cares about 99%. constrictive effect on arterioles à ¯ afterload à heart pumps easier), but loops are
Furosemide
- Beta-1-selective blockers follow the mnemonic: A BEAM of beta-blockers: Atenolol, first-line if the vignette specifically emphasizes the dyspnea and low O2 sats of the
Bisoprolol, Esmolol, Acebutolol, Metoprolol. patient as the focus.
- The important beta-1/2-non-selective blocker is propranolol. - HY point is that they can cause hypokalemia. What USMLE loves to do is tell you a
- Labetalol and carvedilol antagonize both beta-1 and alpha-1. They don’t end in –olol, patient is initiated on furosemide but it is insufficient + now needs a second diuretic à
which makes them easier to remember/group this way. answer = anything that is potassium-sparing (i.e., ENaC inhibitor such as amiloride or
- Beta-1 antagonism slows HR à ­ diastolic filling time à ­ LVEDV. Even though ­ triamterene; or aldosterone receptor antagonist such as spironolactone or eplerenone).
preload ­ oxygen demand, the ¯ chronotropy and inotropy cause a ¯¯ in oxygen - Can also cause ototoxicity (tinnitus, vertigo).
demand, where the net effect is ¯ oxygen demand. - Loops ­ urinary calcium. This is in contrast to thiazides, which ¯ it.
- Step 1 NBME wants beta-blockers as “slowing the rate of diastolic depolarization.” In - Ethacrynic acid is another loop you can be aware of. It is used in patients with sulfa
other words, if HR slows, then fractionally more time is spent in diastole, which means allergies, since furosemide is a sulfa drug. It is also ototoxic.
the process of returning to systole is delayed/protracted. - Hydrochlorothiazide (HCTZ) is HY example; chlorthalidone is “thiazide-like.”
- Labetalol used first-line in patients who have aortic dissection and traumatic rupture of - Inhibit the Na+/Cl- symporter on the apical membrane of the early-DCT.
the aorta. Nitroprusside comes after. - ­ calcium reabsorption in the DCT of the kidney. Used to ¯ recurrence of calcium
- 2CK Q gives “esmolol + nitroprusside” as answer to a traumatic rupture Q, but almost stones by promoting ­ reabsorption of urinary calcium. In turn, they can sometimes
b-blockers always, they will just want “labetalol.” cause hypercalcemia. There is an NBME Q where they ask for MOA of HCTZ, where
- Timolol can be used topically for glaucoma (¯ aqueous humor production). Na+/Cl- symporter inhibition isn’t listed; answer is just “­ calcium reabsorption.”
- Propranolol used for innumerable things à Tx of tachycardia in hyperthyroidism - Thiazides and dihydropyridine calcium channel blockers are used first-line for HTN in
(because also has additional effect of ¯ peripheral conversion of T4 à T3); akathisia patients without any renal or cardiovascular issues. If patient has proteinuria, ­ in
(restlessness as an extra-pyramidal side-effect of antipsychotics); essential, alcoholic, or Thiazides creatinine or renin, or pre-diabetes or diabetes, ACEi or ARBs are used first.
idiopathic tremors (have seen various tremors treated on NBME forms this way, not just - Can be used in heart failure in patients with diabetic nephropathy who are already on
essential tremor); social phobia (if patient has asthma, 2CK Psych form wants benzo ACEi and b-blocker. Sounds specific, but there’s a new 2CK Q that has thiazide as correct
instead); HOCM (¯ HR à ­ LVEDV à ¯ murmur/obstruction). over spironolactone in a diabetic. Spironolactone can cause hyperkalemia in patients
- For propranolol, USMLE wants: ­ LVEDV; ¯ CO, ­ peripheral vascular resistance. who have worrisome kidney function.
- Propranolol is b1/2-non-selective; b2 agonism normally has dilatory effect on - Thiazides can cause gout (i.e., contraindicated in gout).
peripheral arterioles, so if we antagonize à ­ peripheral vascular resistance. - Offline NBME 20 wants you to know thiazides can cause galactorrhea (milky discharge
- For labetalol, USMLE wants: ­ LVEDV; ¯ CO, ¯ peripheral vascular resistance. from the nipples) via some obscure mechanism. Literature search shows it accounts for
- Labetalol, in contrast, has some a1 blockade effect in addition to b1, so PVR is ¯. ~0.2% of adverse effects of thiazides, but count on NBME to ask it.
- Beta-blocker given after ACEi/ARB in Tx of heart failure (i.e., choose ACEi or ARB first). - ENaC inhibitors à block apical sodium channel in cortical collecting duct à ¯ Na+
- Metoprolol used for rate-control in AF. reabsorption à ¯ water reabsorption.
- Beta-blockers can cause depression and sexual dysfunction. - These are potassium-sparing, which means they do not ¯ serum K+. This is because by
- Avoid in patients with lung disease or history of severe or psychotic depression. inhibiting the apical ENaC channel, they indirectly inhibit the basolateral Na+/K+ ATPase
Dobutamine - Beta-1 agonist used in pharmacologic stress testing (i.e., dobutamine-echo). à ¯ Na+ reabsorption + ¯ K+ secretion.
- Potassium channel blocker. Amiloride, - The answer on USMLE for a second diuretic given in a patient already on furosemide
Amiodarone - Can cause TdP, greyish skin discoloration, and thyroiditis. Triamterene who needs additional fluid unloading. Be careful however. I’ve seen one Q where the
- Used for VT in patients without coma or low BP. patient is on furosemide, but the point of the Q is he/she needs HTN control, and the
- Sodium channel blocker. answer is a thiazide, not the ENaC inhibitor. You want to select an ENaC inhibitor
Quinidine
- Can cause TdP and cinchonism (headache + tinnitus). specifically if the Q says, “We have a patient who has peripheral/pulmonary edema due
- Directly blocks myocardial Na+/K+ ATPase pump à causes indirect inactivation of to X cause + is already on furosemide; what do we do now?” à answer = amiloride or
myocardial Na+/Ca2+ ATPase à more Ca2+ remains in myocardial cell à increased triamterene.
Digoxin contractility. - Aldosterone receptor antagonists.
Spironolactone,
- Also has parasympathomimetic effect at nodal tissue that slows HR. - By blocking aldosterone receptor, they ¯ activity of the basolateral Na+/K+ ATPase à
Eplerenone
- In other words, digoxin both slows HR + increases contractility. leads to indirect ¯ activity of apical ENaC à ¯ Na+ reabsorption à ¯ water reabsorption.

MEHLMANMEDICAL.COM 36 MEHLMANMEDICAL.COM 37
MEHLMANMEDICAL.COM MEHLMANMEDICAL.COM

- Potassium-sparing. Same as with ENaC inhibitors, used for fluid unloading in patients - Some sources use 70, rather than 100, for the latter two cutoffs. What I can
who are already on furosemide in whom we worry about dropping their K+ too much. say is that I’ve seen 2CK NBME Qs where they use 100 as the cutoff (i.e., they
- Used in heart failure up the hierarchy of meds – i.e., ACEi/ARB first-line, followed by give an LDL of 95 and statin is wrong, implying LDL is satisfactory).
adding b-blocker, followed by spironolactone (but if patient is diabetic with poor renal - Some sources incorporate a CVD risk % of >7.5%, which is more of a moot /
function, don’t add spironolactone here; give thiazide instead as per new 2CK NBME). pedantic talking point. I’ve seen one 2CK NBME Q where a CVD risk % shows up
- Can be for aldosteronoma (Conn syndrome) prior to surgery. in the stem, but it doesn’t rely on you knowing that point to get it right.
- Spironolactone can cause gynecomastia (anti-androgenic effect by blocking androgen - Fibrates (e.g., fenofibrate) upregulate PPAR-a and lipoprotein lipase; best drugs to
receptors). decrease triglycerides.
- Eplerenone has ¯ risk of gynecomastia compared to spironolactone. - Give if triglycerides >300 mg/dL.
Fibrates
- USMLE-favorite ACE inhibitor. - Can cause myopathy and hepatotoxicity, as well as cholesterol gall stones.
- Prevents conversion of AT-I into AT-II in the lungs. - If the USMLE asks you why statins + fibrates combined have ­ chance of myopathy, the
- Used for HTN in patients with pre-diabetes, diabetes, atherosclerotic disease, or renal answer is “P-450 interaction.”
disease (I talk about this in HY Risk Factors PDF in more detail). - Blocks cholesterol absorption in the small bowel, thereby ¯ LDL.
Lisinopril - Can cause dry cough (ACE is aka bradykininase, so ACEi cause ¯ breakdown of Ezetimibe - USMLE just wants you to know MOA.
bradykinin in lungs à cough). - Doesn’t decrease mortality.
- Can ­ serum K+, since ¯ aldosterone synthesis. Aldosterone normally secretes K+ in the - Bile acid sequestrant. ¯ LDL. Doesn’t ¯ mortality.
distal kidney). - Causes reduced enterohepatic circulation of bile acids at terminal ileum à liver must
Cholestyramine
- Avoid in hereditary angioedema. now convert more cholesterol into bile acids in order to replenish them à liver pulls
- Angiotensin II receptor blocker (ARB). cholesterol out of the blood to accomplish this.
- Use-cases are identical on USMLE to ACEi (i.e., if you see both as answer choices to a Orlistat - Pancreatic lipase inhibitor used for obesity; can cause steatorrhea.
Valsartan
question, they’re usually both wrong because they’re the “same”). - Vitamin B3. Two MOAs USMLE wants you to know: 1) ¯ VLDL export by the liver; 2) ­
- Doesn’t cause dry cough the way ACEi do. HDL more than any other medication.
- a1 agonists à constrict arterioles à ­ BP à HR ¯ due to baroreceptor reflex. - Administration can cause flushing (caused by prostaglandin, so mitigated by asprin),
- Highest yield uses on USMLE are for nasal decongestion à constrict capillaries within gout, and insulin resistance.
Oxymetazoline, nasal mucosa à ¯ inflammation à relief of congestion. Niacin - Deficiency à pellagra à 3Ds à dementia, dermatitis, diarrhea.
Phenylephrine - Can cause rhinitis medicamentosa, which means rebound nasal congestion upon - Can present as delirium instead of overt dementia (biggest risk factor for delirium is
withdrawal if used non-stop for ~5 days. In other words, patients should use only as underlying dementia, so old patient with delirium often has underlying cognitive
needed for a maximum of about ~3-4 days while sick. decline); dermatitis will present on USMLE as either hyperpigmentation of the skin of the
Isoproterenol - b1/2 agonist à increases HR and decreases peripheral vascular resistance. forearms or Casal necklace.
- a2 agonists à prevent presynaptic release of neurotransmitters, especially NE and S. - Sodium channel blocker that can be used in Wolff-Parkinson-White.
Methyldopa, Procainamide
- Methyldopa used for HTN in pregnancy (nifedipine and labetalol also used). - Can cause drug-induced lupus with anti-histone antibodies.
Clonidine
- Clonidine used for various psych treatments (e.g., Tourette). - Na+ channel blocker used as first-line rhythm control in patients fail rate-control for AF.
- a2 antagonist. Flecainide - Patient should have no structural or coronary artery disease. Otherwise use a
Mirtazapine potassium channel blocker like amiodarone or dofetilide.
- Used to treat depression in patients who have anorexia (stimulates appetite).
Ritodrine - b2 agonist used as tocolytic (i.e., slows/delays labor). Dofetilide, - Potassium channel blockers.
- Dilates arterioles à ¯ BP à HR goes ­ to compensate. Ibutilide - Can cause torsades (asked on NBME).
Hydralazine - Used for hypertensive emergencies in pregnancy. Omega/fish oils - Decrease triglycerides.
- Affects calcium currents (but not a calcium channel blocker). - 2CK form gives patient who is in pain from surgery despite being on max doses of
- As discussed earlier, they liberate NO which ­ guanylyl cyclase à ­ cGMP à relaxation morphine + they say blood pressure is elevated + ask what is next best step à answer =
of venous smooth muscle à venous dilation/pooling à ¯ preload on heart à ¯ oxygen “increase bolus of morphine.” Unusual, since they say max dose of morphine already,
demand à relief of anginal pain. but 1) we always full treat pain, and 2) explanation for the Q talks about how pain can
Nitrates - Don’t combine with PDE-5 inhibitors (e.g., sildenafil), which prevent breakdown of lead to high BP.
cGMP. This can lead to severe hypotension. - My point here is that just be aware HTN can be caused by pain à treating pain can
- Combination of hydralazine + nitrates decreases mortality in heart failure. Morphine bring down the BP.
- For sodium nitroprusside, choose arterioles as site of action. - Separately, treatment of MI, acute limb ischemia, and sickle crisis usually includes
- Statins have 2 HY MOAs on USMLE: 1) inhibit HMG-CoA reductase; 2) upregulate LDL oxygen + morphine.
receptors on hepatocytes. - Answer on NBME exam for how a patient using a self-administering pump develops
- Can cause myopathy and toxic hepatitis. An offline NBME has myopathy as correct over morphine overdose = “morphine is converted into active metabolites that accumulate.”
toxic hepatitis. - Otherwise morphine is odd drug for me to put in this table; didn’t know where else to
Statins - Indications for statins on 2CK vary depending on the source (i.e., whether to use the 70 put it though.
vs 100 mg/dL cutoff in certain scenarios), but for IM, give if:
- Age 20-39 if LDL > 190 mg/dL.
- Age 40-75 if LDL > 100 mg/dL.
- Age 20-75 in diabetic if LDL >100 mg/dL.

MEHLMANMEDICAL.COM 38 MEHLMANMEDICAL.COM 39
MEHLMANMEDICAL.COM MEHLMANMEDICAL.COM

Surg Cardio - Endothelin 1 is vasoconstrictor and key mediator in pulmonary


hypertension. USMLE wants you to know this is increased in both cor
pulmonale and left heart failure. Bosentin is an endothelin 1 receptor
Important basic heart failure points for Surgery antagonist.
- Presents as pulmonary findings (i.e., dyspnea, orthopnea, paroxysmal - Nitric oxide synthase, in contrast, USMLE wants you to know is decreased
nocturnal dyspnea). in pulmonary hypertension (makes sense, since NO dilates).
- This is because left-heart problems cause a backup of pressure onto the - A loud P2 and tricuspid regurgitation are HY findings in cor pulmonale. I
pulmonary circulation, leading to increased pulmonary capillary hydrostatic discuss these in more detail in the tables below.
pressure à transudation of fluid into the alveolar spaces (pulmonary
edema). Sometimes this can also cause pleural effusion.
- Left atrial pressure (LAP) = pulmonary capillary wedge pressure (PCWP).
Left heart failure - Therefore, if there is left heart pathology, PCWP is high (exceedingly HY). Bilateral pitting peripheral edema
- Conversely, if a Q gives you normal PCWP, you know there’s nothing wrong - Right heart failure (either due to cor pulmonale or congestive) à ¯ ability to fill right
with the left heart. Qs will often give high PCWP and low BP, where you Cardiac heart à ­ central venous pressure à ­ systemic venous hydrostatic pressure à
need to know immediately that means cardiogenic shock. transudation of fluid from systemic veins/venules into interstitium of legs.
- What USMLE will do is give you some sort of left-heart pathology + - Cirrhosis à ¯ hepatic production of albumin à ¯ intravascular oncotic pressure à
dyspnea, and then ask for the cause of the dyspnea à answer = “increased Hepatic
transudation of fluid from systemic veins/venules into interstitium of legs.
pulmonary capillary hydrostatic pressure.” Another answer in this case is - Proteinuria à hypoalbuminemia à ¯ intravascular oncotic pressure à transudation
“increased alveolar-arteriolar (A-a) oxygen gradient.” Nephrogenic
of fluid from systemic veins/venules into interstitium of legs.
- Presents as systemic findings – i.e., jugular venous distension (JVD) and Drugs - Dihydropyridine calcium channel blockers (i.e., amlodipine, nifedipine).
peripheral edema.
- Strict vegetarianism or veganism à ¯ dietary protein consumption à ¯ intravascular
- Since blood cannot enter the right heart as easily, it backs up to the neck Dietary oncotic pressure à transudation of fluid from systemic veins/venules into interstitium
veins (JVD) and venous circulation (increased hydrostatic pressure in veins
of legs.
à transudation of fluid into legs). The Q might mention that central venous
Pregnancy - A little bit of peripheral edema is normal in pregnancy due to compression of IVC.
Right heart failure pressure is high.
- Hepatosplenomegaly can also be seen in RHF but is very rare on USMLE.
- Normal jugular venous pressure (JVP) is 3cm above the sternal angle. JVD
would be higher than this. Sometimes questions can write that jugular
Unilateral non-pitting edema
venous pulsations are seen 3cm above the sternal angle and the student
- Malignancy (e.g., peau d’orange of breast), Hx of surgery (e.g., mastectomy),
erroneously thinks this means JVD, but this is not the case.
Wuchereria bancrofti (elephantiasis).
- Congestive heart failure = left heart failure + right heart failure.
- Stewart-Treves syndrome is lymphangiosarcoma in the setting of chronic
- The most common cause of right heart failure is left heart failure. Simply Lymphatic insufficiency
lymphatic insufficiency; shows up on Surg form following mastectomy – i.e.,
adding the two together, we now call that congestive heart failure.
Congestive heart failure violaceous lesions on elbow in patient with lymphatic drainage issue ever since
- In congestive heart failure, we’ll see both left- and right-heart failure
a mastectomy many years ago.
findings – i.e., patient will have dyspnea, JVD, and peripheral edema.
- PCWP is elevated in these patients, since the left heart has pathology. - Pretibial myxedema (Graves) à mucopolysaccharide deposition in skin +
surrounding edema.
- Cor pulmonale is defined as right-heart failure due to a pulmonary cause.
- Myxedma (severe hypothyroidism) à despite the name, it refers to general
In other words, the left heart is completely normal in cor pulmonale and
“severe hypothyroidism,” not just skin changes; can cause carpal tunnel
PCWP is normal. Thyroid
syndrome.
- Cor pulmonale will be a patient who has JVD and peripheral edema in the
- “Pretibial myxedema” is only seen in Graves. Paradoxical hyperthyroidism seen
setting of obvious and overt lung disease, such as 100-pack-year smoking
in Hashimoto causing pretibial myxedema is astronomically rare and will get you
history, cystic fibrosis, or pulmonary fibrosis. These can present with lung
questions wrong on USMLE.
findings such as wheezes, where you as the student need to say, “It just
doesn’t seem like they’re focusing on left-heart failure as the cause of the
right-heart failure here. It seems the 100-pack-year smoking Hx causing
Cor pulmonale COPD is why the right heart is failing.”
- The patient can have a “boot-shaped” heart colloquially, which refers to HY Valvular / flow abnormalities on USMLE
right ventricular hypertrophy without left ventricular hypertrophy. - Fixed splitting of S2.
- If the patient has COPD, the massively hyperinflated lungs will push the - Can sometimes be associated with a systolic flow murmur, since more
heart to the midline, causing a long, narrow cardiac silhouette, with a point blood L à R from the LA à RA means more blood flow across the
of maximal impulse in the sub-xiphoid space. pulmonic valve. So Q might say “fixed splitting of S2 and a systolic
- You must know that pulmonary hypertension is the reason the right heart Atrial septal defect murmur.”
decompensates. In both cor pulmonale and congestive heart failure, the - Sometimes can be seen in Qs as “wide, fixed splitting.” I only mention
right heart experiences increased afterload because of pulmonary this because some students get pedantic / ask about this. “Wide splitting”
hypertension. just means right ventricular hypertrophy. So if the Q says “wide, fixed
splitting,” they’re saying the patient has RVH due to an ASD.

MEHLMANMEDICAL.COM 4 MEHLMANMEDICAL.COM 5
MEHLMANMEDICAL.COM MEHLMANMEDICAL.COM

- Patent foramen ovale = ASD on USMLE. Don’t confuse with patent ductus - The mitral valve is replaced if the patient develops severe pulmonary
arteriosus (discussed below). symptoms (i.e., shortness of breath / reduced exercise tolerance), reduced
- ASDs do not need to be repaired unless patient has evidence of ejection fraction, arrhythmia, or endocarditis if valve function is destroyed.
pulmonary hypertension, RVH, or develops an arrhythmia (i.e., usually AF). - Described as “rumbling diastolic murmur with an opening snap”; can also
- If adult with ASD has paradoxical embolus (DVT going through the ASD to be described as “decrescendo mid-late diastolic murmur” (i.e., following
the brain/arterial circulation), this is also indication for repair. the opening snap).
- Holosystolic (aka pan-systolic) murmur at lower left sternal border. - Can cause a right-sided S4 if the pressure backs up all the way to the right
- Can be associated with a diastolic rumble or enlarged left atrium (if more heart (seen on NBMEs sometimes; this confuses students because they
blood going L à R across VSD, then more blood is returning to the LA from think S4 must be LV, but it’s not the case). An S4 is a diastolic sound heard
the lungs à LA dilatation). in either the LV or RV when there is diastolic stiffening due to high
- Seen as part of tetralogy of Fallot (VSD, RVH, overriding aorta, pulmonic afterload.
stenosis). - 99% of mitral stenoses are due to Hx of rheumatic heart disease (i.e., the
- If a VSD is repaired, USMLE wants ­ LV pressure, ¯ RV pressure, and ¯ LA patient had rheumatic fever as a child, where at the time it was mitral
pressure as the changes now seen in the heart. regurg, but years later it has now become mitral stenosis).
- LA pressure ¯ following repair because less blood is circulating through - One 2CK NBME Q mentions patient with history of rheumatic heart
the pulmonary circulation back to the LA. disease who, years later, now has 4/6 rumbling diastolic murmur without
- VSD does not cause cyanosis at birth. Only years later after the higher an opening snap; this is still mitral stenosis. Although opening snap is
blood flow to the lungs results in pulmonary hypertension, followed by buzzy for MS, just be aware it’s not mandatory and that this Q exists on
right ventricular hypertrophy and reversal R à L (Eisenmenger) does the NBME.
Ventricular septal defect patient become cyanotic. - Other HY presentation on USMLE is pregnant women with new-onset
Mitral stenosis
- Murmur can be silent or soft at birth, followed by loud at 7 days of life. dyspnea in 2nd trimester and a diastolic murmur. This is because 50%
The USMLE will ask why the murmur is louder now à answer = decreased increase in plasma volume by 2nd trimester causes the underlying
pulmonary vascular resistance – i.e., the lungs open up during the first subclinical MS to become symptomatic. Don’t confuse this with severe
week of life, resulting in decreased RV pressure and an increase in the L à dyspnea and peripheral edema in late third-trimester, which is instead
R pressure gradient (louder murmur). peripartum cardiomyopathy (antibody-mediated).
- Conversely, if they ask why the murmur was softer at birth compared to - The 1% of MS that’s not due to Hx of RF can be marantic (non-bacterial
now, the answer is “increased pulmonary vascular resistance,” where the thrombotic endocarditis; NBTE) à endocarditis seen due to
lungs were still closed at the time, so there was a lesser gradient L à R hypercoagulable state in the setting of malignancy, where the vegetations
(softer murmur). are small and verrucous, on both sides of the valve. This is in contrast to
- VSDs are repaired if patient develops pulmonary hypertension, RVH, bacterial endocarditis, which causes large, floppy vegetations that lead to
arrhythmia, Eisenmenger syndrome, recurrent endocarditis (turbulence of MR, not MS.
blood due to VSD can ­ risk of valve infections), or aortic regurgitation (if - Libman-Sacks endocarditis seen in SLE is due to antiphospholipid
VSD located near the aortic valve). antibodies and is a type of NBTE.
- Seen in Down syndrome. - Balloon valvuloplasty is the 1st-line Tx for mitral stenosis. This is done if
Atrioventricular septal defect - Between the atrium and ventricle, aka “endocardial cushion defect,” patient has minimal calcification of the valve + has pulmonary HTN.
although this latter term can also apply to ASD and VSD in Downs. - Mitral valve replacement is done if balloon valvuloplasty fails, if patient
- Holosystolic (pan-systolic) or just regular “systolic,” 29 times out of 30. has severe MS with dyspnea, arrhythmia, or calcification of the valve.
- Q on old NBME has MR as “mid-systolic,” but I contend this is erratum. - Most common murmur.
- Most USMLE questions will not mention it radiating to the axilla. - Described as mid-systolic click.
- Highest yield cause of MR on USMLE is post-MI papillary muscle rupture. - “Myxomatous degeneration” is buzzy term that refers to connective
USMLE is obsessed with this. They’ll say hours to days after an MI, patient tissue degeneration causing MVP in Marfan and Ehlers-Danlos.
has new-onset systolic murmur à answer = MR. - Almost always asymptomatic. On 2CK forms, they want you to know
- Seen acutely in rheumatic heart disease (valve scars over years later and about “mitral valve prolapse syndrome,” which is symptomatic MVP that
becomes mitral stenosis). presents as repeated episodes of “fleeting chest pain” on the left side in an
- Can be caused by general ischemia / dilated cardiomyopathy. otherwise healthy patient 20s-30s. They might say there is Hx of MI in the
Mitral regurgitation - Can cause JVD (i.e., back up all the way to the right heart); this is asked family, but this is MVPS, not MI. Answer on surgery form is “no treatment
multiple times on the new NBMEs. necessary." Beta-blockers can be used in theory to help with MVP, but this
Mitral valve prolapse
- Surgery forms want you to know that you do not do preoperative stress is a wrong answer on that Q.
tests to determine perioperative MI risk if the patient has mere mitral - USMLE loves using MVP as a distractor in panic disorder questions. They
regurg without other risk factors. For example, one of the Surg forms gives will give long paragraph about panic attack/disorder + also mention there’s
a Q where smoker with MR has no shortness of breath or chest pain with a mid-systolic click; they’ll ask for cause of patient’s presentation à
exertion, and the answer is “no further management indicated,” where answer = panic disorder, not MVP à student is confused because they say
exercise stress test is wrong. mid-systolic click, but the MVP isn’t the cause of the patient’s
- If the patient has Sx of heart failure or ischemia, then we do pre-op stress presentation; the panic disorder is; MVP’s are usually incidental, benign,
test to determine MI risk. I discuss stress tests later in this chapter. and asymptomatic.
- MVP does not progress to mitral regurg almost always. So don’t think
that MVP and MR are the same.

MEHLMANMEDICAL.COM 6 MEHLMANMEDICAL.COM 7
MEHLMANMEDICAL.COM MEHLMANMEDICAL.COM

- Decrescendo holo-diastolic (pan-diastolic) murmur; can also be described - In theory, would be a rumbling diastolic murmur similar to mitral
as “early diastolic murmur,” or “diastolic murmur loudest after S2.” stenosis, but would increase with inspiration since it’s on the right side of
- Causes wide pulse pressure (i.e., big difference between systolic and the heart.
diastolic pressures, e.g., 160/50, or 120/40) à results in head-bobbing and - Same as with tricuspid stenosis, this is a nonexistent murmur on USMLE.
bounding pulses (don’t confuse with slow-rising pulses of aortic stenosis). I’ve never seen it assessed.
Pulmonic regurgitation
- The bounding pulses can be described on NBME as “brisk upstroke with In theory it would be the same as aortic regurg but on the right (i.e., holo-
precipitous downstroke.” In turn, they can just simply say, “the pulses are diastolic murmur), but increases with inspiration.
brisk,” meaning the systolic component is strong. - Seen in tetralogy of Fallot.
- I would say 4/5 times bounding pulses means AR. The other 1/5 will be - Described as mid-systolic murmur, or just regular “systolic” murmur, that
PDA and AV fistulae (discussed below). Bounding pulses occur when blood increases with inspiration, at the left sternal border, 2nd intercostal space.
Pulmonic stenosis
Aortic regurgitation quickly leaves the arterial circulation. In AR, the blood quickly collapses out This is the theoretical location, whereas AS is the 2nd intercostal space on
of the aorta back into the LV. In PDA, it leaves the aorta and enters the the right, not left. But the USMLE often isn’t strict about murmur locations
ductus arteriosus; in AV fistulae, it leaves for a vein. this way.
- Highest yield cause on USMLE is aortic dissection à can retrograde - Murmur described three ways on USMLE: 1) continuous, machinery-like
propagate toward the aortic root causing aortic root dilatation and AR. murmur; 2) pan-systolic pan-diastolic murmur (meaning it’s continuous
- Even though MVP is most common in Marfan and Ehlers-Danlos, AR is throughout both systolic and diastole); and 3) to-and-fro. The latter shows
second most common in these patients, since if they get aortic dissection, up on offline 2CK NBME.
this can lead to AR. - Classically associated with congenital rubella (HY). They’ll give a kid born
- Can lead to volume overload on the LV and eccentric hypertrophy. with a PDA and then ask what the mom experienced while pregnant;
- Aortic valve is replaced if patient has EF <50%, there is significant left answer = arthritis and/or rash (rubella often presents as arthritis in adults).
ventricular dilatation, or if severe endocarditis has obliterated the valve. Patent ductus arteriosus
- Indomethacin (NSAID) will close the PDA.
- Mid-systolic murmur, or just “systolic” murmur; can also be described as (PDA)
- Prostaglandin E1 is used to keep a PDA open (if a kid with congenital
“late-peaking systolic murmur with an ejection click.” heart malformations is born cyanotic and we need to buy time until
- Radiates to the carotids. This descriptor shows up quite frequently on surgery).
NBME (way more than radiation to the axilla for MR). - An open PDA can mask cyanosis in a newborn in a variety of conditions
- Causes slow-rising pulses, aka “pulsus parvus et tardus” (don’t confuse (i.e.,., hypoplastic left heart syndrome or pre-ductal coarctation). If they
with bounding pulses of AR). tell you a kid is born with normal APGAR scores but a week later becomes
- SAD à Syncope, Angina, Dyspnea; classic combination seen in AS, albeit cyanotic and they ask why, the answer is “closure of ductus arteriosus.”
not mandatory. If you get a question where they say systolic murmur but - 1) Pulmonic stenosis; 2) RVH; 3) overriding aorta; 4) VSD.
you’re not sure of the diagnosis, if they say chest pain or fainting, you - If you’re asked which component most determines prognosis, the answer
Aortic stenosis
know it’s AS. is the degree of pulmonic stenosis.
- Often caused by bicuspid aortic valve. The patient need not have Turner - The child will not be cyanotic at birth, but then years later, will develop
syndrome and often won’t. Bicuspid valve is usually inherited as an Eisenmenger syndrome (i.e., a reversal of the LàR shunt over the VSD to
autosomal dominant familial condition. be RàL) and cyanosis, where the stem gives a school-age kid who squats
- The bicuspid valve need not calcify in middle-age prior to the AS forming. Tetralogy of Fallot
on the playground to relieve symptoms.
I’ve seen it cause AS in high schooler and in a baby on NBME. - Squatting ­ afterload, which ­ LV pressure, which ¯ the pressure
- Do aortic valve replacement on 2CK if 1) cross-section of valve is <1.0 gradient of the RàL shunt, thereby mitigating cyanosis.
cm2, or 2) there is SAD. They ask both of these as separate Qs where they - Squatting also ­ preload by ­ venous return back to the right heart. But it
want valve replacement. is the effect of ­ afterload that is most related to the ¯ in symptoms.
- Will be described on USMLE as a holosystolic murmur that increases with - Tx is surgical correction in infancy or early childhood.
inspiration. - Systolic murmur seen in the setting of higher heart rate caused by
- Right-sided heart murmurs get worse with inspiration à diaphragm infection, anemia, or pregnancy. Caused by increased flow across the
moves down à decreased intra-thoracic pressure à increased right-heart pulmonic and/or aortic valves.
filling. - Known as a functional murmur because this means it goes away once the
- Can cause pulsatile liver. heart rate comes back down.
- Highest yield cause of TR on USMLE is pulmonary hypertension / cor - Seen all over 2CK Peds forms, where they try to trick you into thinking the
Tricuspid regurgitation pulmonale. I see this all over the NBME exams. For whatever reason, these Functional (flow) murmur
kid has a valvular pathology of some kind, but there isn’t; there will merely
conditions do not cause pulmonic regurg; they cause tricuspid regurg. In be an infection or simple viral infection.
other words, if you see tricuspid regurg in a Q, your first thought should be - Can be seen sometimes with ASD, where the patient will have fixed
pulmonary hypertension or cor pulmonale (right heart failure due to a splitting of S2 “plus a systolic murmur” à merely higher right-sided
pulmonary cause). volume, so more flow across the pulmonic valve.
- IV drug user endocarditis is obvious risk factor for TR, but weirdly - No Tx necessary.
nonexistent on USMLE. - On 2CK Peds form; described as a murmur in the neck that abates when
- Carcinoid syndrome is theoretical cause but lower yield. Venous hum the kid is laid supine + the neck rotated.
- Nonexistent murmur on USMLE. I don’t think I’ve ever seen this assessed - Benign + don’t treat.
Tricuspid stenosis
once on any NBME exam for Steps 1 and 2 combined.

MEHLMANMEDICAL.COM 8 MEHLMANMEDICAL.COM 9
MEHLMANMEDICAL.COM MEHLMANMEDICAL.COM

- Associated with cardiac tumors (i.e., myxoma in adult, or rhabdomyoma - Wide splitting of S2, right-axis deviation on ECG, and right bundle branch
in kids for tuberous sclerosis). block (RBBB) all = right ventricular hypertrophy on USMLE.
“Ball-in-valve” murmur
- Described as a diastolic rumbling murmur that abates when the patient is - Means left ventricular hypertrophy on USMLE.
re-positioned unconventionally (e.g., onto his or her right side). - A2 occurs after P2 (normally we have A2 before P2).
- Aka persistent fetal circulation. - Left ventricular pressure is high and A2 delayed to the point that it
Paradoxical splitting of S2
- Q will give a post-term birth at 42 or 43 weeks + meconium-stained fluid actually occurs on the opposite side of P2.
+ echo of the neonate shows a RàL shunt across the foramen ovale. - Paradoxical splitting of S2, left-axis deviation on ECG, and left bundle
Student says, “Wait, but isn’t the foramen ovale between the atria, and branch block (LBBB) all = left ventricular hypertrophy on USMLE.
Persistent fetal hypertension that’s only open in the fetus but is supposed to close after birth?” Correct. - Refers to narrowing of the aortic arch (this is referred to as coarctation;
Hence we have persistent fetal circulation. do not use the word stenosis to describe this).
- Answer on USMLE will be “failure of pulmonary vasodilation.” Meconium - Classically seen in Turner syndrome, but absolutely not mandatory.
aspiration syndrome can ¯ opening of the lung vasculature, leading to ­ Shows up idiopathically in plenty of NBME Qs. I point this out because
right heart pressure and ­ risk of persistent fetal circulation. students often think the patient must have Turner syndrome.
- Both are diastolic sounds. - Presents as upper extremities that have higher BP, brisk pulses, and are
- S3 is due to high volume/preload in the left ventricle, causing a warmer; the lower extremities have lower BP, weak pulses, and are cooler.
reverberation against the wall. Coarctation of the aorta - Sometimes the Q can just say, “the radial pulses are brisk.” à The
- S3 can sometimes be physiologic (i.e., normal / no problem) in pregnancy implication is, “Well if they’re saying specifically that the radial pulses are
and high-endurance athletes. Patient will have eccentric hypertrophy brisk, that must mean the pulses in the legs aren’t.”
(sarcomeres laid in linear sequence). If pathologic, it is due to dilated - Murmur sound not important for USMLE. Can sometimes be described as
cardiomyopathy with reduced ejection fraction (<55%) or high-output a systolic murmur heart in the infrascapular region.
cardiac failure (EF >70%). - Can cause LVH with left-axis deviation ECG (on NBME).
- There is one question on IM CMS form 7 where they give an S3 in - USMLE doesn’t give a fuck about pre- vs post-ductal. Pre-ductal in theory
diastolic dysfunction. I’m convinced this is an erratum, but I need to will be a very sick neonate. Post-ductal will be an adult (most cases).
mention it because it exists on the NBME form. - Confusing condition when you’re first learning things.
- S4 is due to high pressure/afterload on the left (but sometimes right) - The vertebral artery (goes to brain) is the first branch of the subclavian
S3 versus S4 ventricle, causing a stiffened ventricle with diastolic dysfunction and artery (goes to arm).
concentric hypertrophy (sarcomeres laid in parallel). It is always - If there is a narrowing/stenosis of the proximal subclavian prior to the
pathologic. It is usually caused by systemic hypertension causing afterload branch point of the vertebral artery, this can lead to lower pressure in the
on the LV, or aortic stenosis. vertebral artery.
- S4 can sometimes be right-sided on USMLE. There is a 2CK Q where they - This can cause a backflow of blood in the vertebral artery, producing
give severe mitral stenosis and say there’s an S4, but it’s for the RV not LV. miscellaneous neuro findings such as dizziness.
Some weird/annoying points: - Blood pressure is different between the two arms.
Subclavian steal syndrome
- The combo of S3 and S4, seen together in the same vignette, can be seen - USMLE will ask the Q one of two ways: 1) they’ll give you dizziness in
in high-output cardiac failure. For example, they will say a patient as an AV someone who has BP different between the arms and then ask for merely
fistula/conduit, or has Paget disease, and they will say there’s S3 and S4 “subclavian steal syndrome,” or “backflow in a vertebral artery” as the
and ask for diagnosis à answer = high-output cardiac failure. The take- answer. Or 2) they’ll give you BP in one of the arms + give you dizziness,
home point is that high-output failure can present with either an isolated then the answer will be, “Check blood pressure in other arm.”
S3 or the combo of S3 and S4 together, but never S4 alone on USMLE. - Next best step in Dx is CT or MR angiography (asked on 2CK NBME).
- One of the highest yield cardiac sounds on USMLE, almost always - I should point out that probably 3/4 questions on USMLE where blood
overlooked by students. pressure is different between the arms, this refers to aortic dissection. But
- Means pulmonary hypertension or cor pulmonale on USMLE. 1/4 is subclavian steal syndrome. As per my observation.
- The pulmonic valve slams shut due to high pressure distal to it. - Presents same as subclavian steal syndrome with otherwise unexplained
- For example, they’ll give a smoker who simply has a loud P2 à this just dizziness, but blood pressure is not different between the arms because
means patient has pulmonary hypertension. Not complicated. the subclavian is not affected.
Vertebral artery stenosis
Loud P2 - Also recall that I said above that highest yield cause of tricuspid regurg on - Caused by atherosclerosis. CT or MR angiography can diagnose.
USMLE is pulmonary hypertension / cor pulmonale. So both what I want - “Vertebrobasilar insufficiency” is a broader term that refers to patients
you to remember is both TR and loud P2 for this. who have either subclavian steal syndrome or vertebral artery stenosis.
- Sometimes the UMSLE will just say “loud pulmonic component of S2,” or - 2CK Neuro forms assess vertebral artery dissection, where they want you
“loud S2,” rather than saying “loud P2.” I’ve never seen “loud A2” on to know a false lumen created by dissection in a vertebral artery can lead
USMLE, but in theory this means systemic hypertension. to stasis and clot formation, which in turn can embolize to the brain and
- A soft P2 refers to pulmonic stenosis, but is LY. cause stroke.
Vertebral artery dissection
- Means right ventricular hypertrophy on USMLE. - NBME can mention recent visit to a chiropractor (neck manipulation is
- A2 and P2 are far apart. known cause).
Wide splitting of S2 - You don’t have to worry about the mechanism. But in short, the more - The answer on the NBME is heparin for patients who have experienced
pressure you have in a ventricle, the more delayed the semilunar valve will posterior stroke due to vertebral artery dissection. Sounds weird because
close. So if we have RVH, P2 occurs later, widening the split.

MEHLMANMEDICAL.COM 10 MEHLMANMEDICAL.COM 11
MEHLMANMEDICAL.COM MEHLMANMEDICAL.COM

it’s arterial, but it’s what USMLE wants. Take it up with them if you think once again need not mention carotid bruit; apparently it is not a sensitive
it’s weird. finding (i.e., we cannot rule-out ­ occlusion just because we don’t hear it).
- Shows up on 2CK form as patient with stroke-like presentation + who - As discussed above in the aortic regurg section, USMLE loves this as most
simultaneously has ipsilateral facial/neck pain. common cause of AR due to retrograde propagation toward the aortic
Carotid artery dissection - The pain is due to stretching of nociceptors secondary to vascular root. For example, patient with Hx of HTN, cocaine use, or a connective
dilation. tissue disorder (i.e., Marfan, Ehlers-Danlos) who has a diastolic murmur,
- Stasis within false lumen can lead to embolus to brain/eye. you should be thinking immediately that this is dissection.
- Caused by atherosclerosis. - “Medial necrosis” is a term that is used on NBME exams to describe
- HTN biggest risk factor for atherosclerosis specifically of the carotids changes to the aorta in dissection. In the past, “cystic medial necrosis”
(strong systolic impulse pounds the carotids à endothelial damage à used to be buzzy for dissection due to Marfan syndrome, but I haven’t
atheromatous plaque formation). seen USMLE care about this. I have, however, seen a dissection Q on
- Carotid bruit only seen in about 25% of Qs. Don’t rely on this as crutch. NBME where it is due to hypertension, and simply “medial necrosis” is the
- Vignette will give a stroke, TIA, or retinal artery occlusion in the setting of answer.
a patient with HTN. à You have to be able to make the association that a - As mentioned above, 3/4 Qs where BP is different between the arms
plaque from one of the carotids has launched off, since HTN = ­ risk. refers to aortic dissection. A Q on 2CK IM form 7 has “thoracic aortic
- USMLE will then ask for management (2CK only): dissection” where not only is the BP different between the arms, but it’s
- Do carotid duplex ultrasonography as next best step in diagnosis to look also different between the L and R legs (i.e., L-leg BP is different from R-leg
for degree of occlusion. I’ve never seen carotid angiography as a correct BP) à sometimes thoracic aortic dissections can anterograde propagate
answer on NBME exams. all the way down to the abdominal aorta.
- If occlusion >70% symptomatic, or >80% asymptomatic, then do Aortic dissection
endarterectomy. “Symptomatic” = stroke, TIA, or retinal artery occlusion.
A mere bruit is not a symptom; that is a sign.
- If under these thresholds, do medical management only, which requires a
triad of: 1) statin; 2) ACEi or ARB; and 3) anti-platelet therapy.
- The USMLE will not force you to choose between low- and high-potency
statins.
- USMLE tends to list lisinopril as their favorite ACEi for HTN control.
- It’s to my observation aspirin alone is sufficient on NBME exams for anti-
platelet therapy, even though in real life patient can receive either aspirin
alone; the combo of aspirin + dipyridamole; or clopidogrel alone.
- USMLE will not give borderline carotid occlusion thresholds – i.e., they’ll
Carotid artery stenosis say either 30% or 90%. If they list the % as low, look at the vignette for the
drugs they list the patient on. Sometimes they’ll show the patient is
already on statin, lisinopril, and aspirin, and then the answer is just - You do not need to memorize these aortic aneurysm types. I’m just
“continue current regimen.” I have once seen “add clopidogrel” as a wrong showing you that if the common iliacs are involved (as with left image), BP
answer in this setting, which makes sense, since the combo of aspirin + can differ as well between the legs.
clopidogrel is never given anyway. - Tx for ascending aortic aneurysm (type A) = labetalol + surgery.
- Sometimes they will give you a low carotid occlusion % + say the patient - Tx for descending aortic aneurysm (type B) = labetalol alone initially.
is on 2 of 3 drugs in the triad, and then the answer is just “add aspirin,” or - Caused by deceleration injury. Most common cause of death due to car
“add statin,” or “add lisinopril.” accident or fall. Exceedingly HY on 2CK.
- If the vignette doesn’t mention elevated BP but says you have some - Will be described as patient following an MVA who has “widening of the
random dude over 50 with a stroke, TIA, or retinal artery occlusion, the mediastinum.” They’ll then ask for the next best step à answer = aortic
next best step is carotid ultrasonography to look for carotid stenosis. In angiography (aka aortography), OR CT angiography.
other words, it is assumed the patient has a carotid plaque in this setting. - New 2CK form has “CT scan of the chest” straight up as the answer,
- If the vignette gives patient with episodes of unexplained syncope or which refers to CT angiography. NBME/USMLE will not force you to choose
light-headedness, but not stroke, TIA, or retinal artery occlusion, then the between aortography or CT angiography; they’ll just list one.
Traumatic rupture
next best step is ECG, followed by Holter monitor, looking for atrial - Labetalol used first-line in patients who have aortic dissection and
of the aorta
fibrillation (AF causes LA mural thrombus that launched off to brain/eye). traumatic rupture of the aorta. Nitroprusside comes after.
- The triad of 1) statin; 2) ACEi or ARB; and 3) anti-platelet therapy is also - Labetalol is answer on NBME even in patient who has low BP due to
done for general peripheral vascular disease unrelated to carotid stenosis rupture or dissection due to the drug ¯ shearing forces. I’ve seen students
(i.e., if a patient has intermittent claudication). get this wrong saying, “But patient has low BP though.” My response is, file
- Stroke, TIA, or retinal artery occlusion, if they don’t mention HTN, but a complaint with the exam not with me.
they mention an abdominal bruit, you will still do a carotid duplex - 2CK Q gives “esmolol + nitroprusside” as answer to a traumatic rupture
ultrasound. The implication is that the bruit in the abdomen could be a Q, but almost always, they will just want “labetalol.”
AAA or RAS, where atherosclerosis in one location means atherosclerosis - Emergency surgical repair is indicated following IV drug administration.
everywhere, so the patient likely has carotid stenosis by extension. They Aortic aneurysm - Can present as “visible pulsation” on USMLE.

MEHLMANMEDICAL.COM 12 MEHLMANMEDICAL.COM 13
MEHLMANMEDICAL.COM MEHLMANMEDICAL.COM

- For aortic aneurysm, they can say “visible pulsation above the
manubrium,” or “pulsatile mass above the manubrium.” There can also be
a tracheal shift. I’ve seen students select pneumothorax here. But for
whatever reason you can get tracheal shift in thoracic aortic aneurysm. For
AAA, there can be “visible pulsation in the epigastrium.”
- Biggest risk factor for AAA is smoking.
- Do a one-off abdominal ultrasound in both men and women 65+ who are
ever-smokers. This screening used to be just performed on men, but now
it includes women.
- For Surgery, AAA repair is indicated if the aneurysm is >5.5 cm or the rate
of change of size increase is >0.5cm/month for 6 months. This is on
Surgery form, where they give a patient with a 4-cm AAA and ask why
serial ultrasounds are indicated à answer = “size of aneurysm.”
- In general, perioperative MI risk is assessed using a pre-op stress test.
2CK NBME Q has dipyridamole and thallium pharmacologic stress test as
answer in patient with 6-cm AAA prior to surgery.
- Diabetes is protective against aneurysm. Non-enzymatic glycosylation of
endothelium causes stiffening of the vascular wall. - Surg form shows obscure image similar to above (without the arrow) +
- Don’t do AAA repair on USMLE in patient who has advanced they tell you there’s continuous murmur à answer = “size of lesion.”
comorbidities or terminal disease, e.g., stage 4 lung cancer. - Another NBME Q gives 45-year-old male will nosebleeds since
- 2CK Surg loves “pulsatile hematoma” in the neck in trauma patients, adolescence + S3 heart sound + dyspnea + they show you pic of tongue;
where the answer is “endotracheal intubation.” Sounds nitpicky, but they ask for the cause of dyspnea.
shows up repeatedly.
- Can be idiopathic, iatrogenic (i.e., dialysis), from injury (i.e., stab wound),
or caused by other disease (i.e., hereditary hemorrhagic telangiectasia or
Paget disease of bone).
- Similar to aortic aneurysms, AV fistulae can sometimes present with
pulsatile mass, but in a weird location, e.g., around the left ear in patient
with tinnitus (on NBME exam). Student says, “Why is it at the left ear
though?” à No fucking idea. Take it up with NBME.
- Highest yield point is they can cause high-output cardiac failure. This is
because blood quickly enters the venous circulation from the arterial
circulation à combo of ­ preload back to right heart + poorer arterial
perfusion distal to the fistula à compensatory ­ CO.
- AV fistulae can sometimes present with a continuous machinery murmur
Arteriovenous fistula
similar to a PDA, since blood is continuously flowing through it. They
might say a continuous machinery-like murmur is auscultated in the leg at
site of prior stab wound.
- As discussed earlier, they can present with bounding pulses similar to AR.
- Student says, “Well how am I supposed to know if it’s AV fistula then if it - Answer = “Pulmonary arteriovenous fistula” (leading to high-output
sounds like other conditions too?” à by paying attention to HY points like, failure); diagnosis is hereditary hemorrhagic telangiectasia. USMLE will
“Is there lone S3 or S3/4 combo or EF >70%? Is there Hx of penetrating basically always show you a pic of red dots on the tongue/mouth or finger
trauma? Or does the patient have Paget? Etc.” in a patient with nosebleeds.
- 2CK Surg Q shows you obscure angiogram of a fistula in the leg + tells you - Likewise, be aware intraosseous AV fistulae can occur in Paget, as
there’s a continuous machinery murmur; they ask what most likely mentioned before.
determines prognosis in this patient à answer = “size of lesion.”

MEHLMANMEDICAL.COM 14 MEHLMANMEDICAL.COM 15
MEHLMANMEDICAL.COM MEHLMANMEDICAL.COM

HY Murmur / ECG points for Surg sometimes AF can trigger “rapid ventricular response,” where HR goes >150 and
- Described as “irregularly irregular” rhythm. low BP can occur.
- Has classic sawtooth appearance.

Atrial flutter

- Notice how the QRS complexes are at random and irregular distances from
one another. This is the “irregularly irregular” pattern. - Low yield for USMLE. I think it’s asked once on a 2CK NBME. But as student
- AF is hugely important because it can cause turbulence/stasis within the left you should know it exists / the basic ECG above.
atrium that leads to a LA mural thrombus formation. This thrombus can launch - Causes wide-complex QRS complexes (>120 ms; normal is 80-120 ms).
off (i.e., become an embolus) and go to brain (stroke, TIA, retinal artery
occlusion), SMA/IMA (acute mesenteric ischemia), and legs (acute limb
ischemia).
- AF HY in older patients, especially over 75. Vignette will usually be an older
patient with a stroke, TIA, or retinal artery occlusion, who has normal blood
pressure (this implies carotid stenosis is not the etiology for the embolus).
- AF usually is paroxysmal, which means it comes and goes. The vignette might
say the patient is 75 + had a TIA + BP normal + ECG shows sinus rhythm with no
abnormalities à next best step is Holter monitor (24-hour ambulatory ECG
monitor) to pick up the paroxysmal AF (e.g., when the patient goes home and
has dinner).
- After AF is diagnosed with regular ECG or Holter, 2CK wants echocardiography - Exceedingly HY for 2CK that you know VT is wide-complex, whereas SVT is
as the next best step to visualize the LA mural thrombus. narrow-complex. If you look at above ECG, even if you say, “No idea what I’m
- Patient who has severe abdominal pain in setting of AF or hyperthyroidism looking at.” You can tell the complexes look wide like mountains in comparison
Atrial fibrillation (AF) (which can cause AF), diagnosis is acute mesenteric ischemia; next best step is to a typical ECG.
mesenteric angiography; Tx is laparotomy if unstable (answer on NBME). - VT is treated with anti-arrhythmics – i.e., amiodarone. If patient has coma or
- Severe pain in a leg + absent pulses in patient with irregularly irregular rhythm Ventricular tachycardia hemodynamic instability (low BP), the NBME answer is direct current
= acute limb ischemia; USMLE wants “embolectomy” as answer. (VT) countershock or cardioversion (same thing).
- Any structural abnormality of the heart, either due to LV hypertrophy, - Premature ventricular complex (PVC) is asked on 2CK.
ischemia, growth hormone/anabolic steroid use, prior MI, etc., can lead to AF.
- For 2CK, you need to know AF patient will get either aspirin or warfarin. This is
determined by the CHADS2 score. There are variations to the score, but the
simple CHADS2 suffices for USMLE à CHF, HTN, Age 75+, Diabetes,
Stroke/TIA/emboli. Each component is 1 point, but stroke/TIA/emboli is 2
points. If a patient has 0 or 1 points, give aspirin; if 2+ points, give warfarin. This
is important for 2CK.
- “Emboli” refers to Hx of AF leading to stroke, TIA, acute, mesenteric ischemia,
or acute limb ischemia – i.e., any Hx of embolic event. 2CK IM form 7 gives short
vignette of 67F with chronic AF + Hx of acute limb ischemia + no other info
relating to CHADS, and answer is warfarin to prevent recurrence; aspirin is
- Note on the above strip, we have a wide complex (meaning ventricular in
wrong.
origin) that occurs earlier (hence premature). What they do on the NBME is
- Some students will ask about NOACs, e.g., apixaban, etc., for non-valvular AF
show you this strip and ask where this abnormality originates from, then the
à I’ve never seen NBME care about this stuff. They seem to be pretty old-
answer is just “ventricle.”
school and just have warfarin as the answer, probably because there isn’t
- Don’t treat PVCs on USMLE.
debate around whether it can be used; use of NOACs is less textbook.
Supraventricular - Causes narrow / needle-shaped complexes. Make sure you’re able to contrast
- AF patient should also be on rate control before rhythm control. The USMLE
tachycardia this with VT above, which is wide-complex.
actually doesn’t give a fuck about this component of management, although in
(SVT)
theory metoprolol or verapamil is standard. You could be aware for Step 3 that
flecainide is first-line for rhythm control if patients fail rate-control and have a
structurally normal heart and no coronary artery disease.
- 2CK form has “electrical cardioversion” as the answer for patient with AF who
has hemodynamic instability (i.e., low BP). What you need to know is:

MEHLMANMEDICAL.COM 16 MEHLMANMEDICAL.COM 17
MEHLMANMEDICAL.COM MEHLMANMEDICAL.COM

- Post-MI papillary muscle rupture resulting in mitral regurg is exceedingly HY.


As discussed earlier, if patient has MI followed by new-onset systolic murmur
hours to days later, with or without dyspnea, that’s mitral regurg.
- Stroke-like presentation in patient who had MI weeks ago à “embolus from
ventricular septal aneurysm” (on 2CK Neuro form).
- Most common cause of death due to MI is ventricular fibrillation (VF).
- Fibrosis of myocardium in the months-years post-MI increases risk of
arrhythmias such as AF, SVT, VT, etc. There’s no specific arrhythmia you need to
memorize. Just know the risk is there in the future.
- Q waves on an ECG mean old MI / history of MI. The vignette might give you
patient who has light-headedness / fainting + they say patient has Q waves in II,
III, aVF, and the answer will be something like “paroxysmal supraventricular
tachycardia.” Student thinks this specific arrhythmia matters, but it doesn’t. The
- Notice the complexes are narrow / look like needles. This means the tachy
point is that Hx of MI means patient is at risk for nearly any arrhythmia now.
originates above the ventricles (hence SVT).
- MI classically causes coagulative necrosis of the myocardium.
- Treatment of SVT exceedingly HY on 2CK.
- With cardiogenic shock as a result of MI, the arrows USMLE wants are: ¯
- First step is carotid massage (aka vagal maneuvers). In pediatrics, they can do
icepack to the face. cardiac output, ­ peripheral vascular resistance, ­ PCWP.
- If the above doesn’t work, the next step is give adenosine (not amiodarone). - MI can lead to acute tubular necrosis from cardiogenic shock à acute drop in
- Same as with VT, if the patient has coma or low BP, shocking the patient is the renal perfusion. This is not pre-renal. I discuss this in detail in the renal section.
first step. In other words, for both SVT and VT, you must shock first in the - First treatment for MI is aspirin. After aspirin is given, the next drug to give is
setting of coma or hemodynamic instability. It’s for stable SVT and VT that the clopidogrel (an ADP P2Y12 blocker) as dual anti-platelet therapy.
treatments differ on USMLE. - USMLE wants you to know anyone with acute coronary syndrome (i.e., MI or
unstable angina) gets coronary catheterization. This is answer on new 2CK
- Will present as ST-elevations in 3-4 contiguous leads.
NBME exam.
- It’s to my observation that more extensive management of MI on USMLE, such
as use of beta-blockers, nitrates, morphine, oxygen, statin, percutaneous
coronary intervention, etc., isn’t assessed in detail. I can comment, however,
that one 2CK Q wants you to know nitrates are contraindicated in right-heart
MIs, which includes inferior MI in most people due to the right coronary
supplying the PDA. This is because right-sided MIs are preload-dependent,
which means they need sufficient preload to maintain BP.
- Percutaneous coronary intervention (PCI) is done in patients with STEMIs
within 90 minutes of reaching hospital.
- Shows up on ECG as diffuse ST-elevations (i.e., in all leads rather than 3-4
contiguous leads as with MI). PR depressions can also be seen, but I’ve never
seen the USMLE give a fuck about the latter.
- Patient will have pain that’s worse when lying back, better when leaning
forward. In turn, the patient can present walking through the door bent over at
the waist.
Acute MI (STEMI) - Serous pericarditis will be post-viral, secondary to autoimmune disease, or due
to cocaine use.
- NBME Q gives pericarditis + a bunch of different organism types (i.e., fungus,
- The above is an inferior MI, as evidenced by ST-elevations in leads II, III, and bacterium, etc.), and answer is “virus.”
aVF. The answer for the affected vessel is the posterior descending artery (PDA - Patient with rheumatoid arthritis or SLE notably at risk for pericarditis. In other
supplies the diaphragmatic surface of the heart); since >85% of people have Pericarditis words, don’t get confused if they mention pericardial friction rub in vignette of
right-dominant circulation (meaning the PDA comes of the right main coronary), RA or SLE; this is common.
sometimes the answer for inferior MI can just be “right coronary artery.” - For cocaine use, they’ll say a 22-year-old male has chest pain after a night of
- If the Q says left-dominant circulation, the sequence USMLE wants is: left main heavy partying + ECG shows diffuse ST-elevations à Dx = pericarditis.
coronary à left circumflex à PDA. - Uremic pericarditis is HY for 2CK. Q will give ultra-high creatinine and BUN and
- The apex of the heart is supplied by the left anterior descending artery (LAD). say there’s a friction rub à treatment = hemodialysis.
If there are ST-elevations in leads V1-V3, choose LAD as the answer. - Treatment for pericarditis is same as acute gout à NSAIDs, colchicine,
- The left-lateral heart is supplied by the left circumflex artery. If there are ST- steroids.
elevations in leads V4-V6 for lateral MI, choose left circumflex. - Fibrinous pericarditis is post-MI and occurs as two types: 1) literally “post-MI
- Reciprocal ST-depressions in the anterior leads V1-V3 can reflex posterior wall fibrinous pericarditis,” which will simply be friction rub within days of an MI; 2)
MI (i.e., we have “elevations” out the back of the heart, so they look like Dressler syndrome (antibody-mediated fibrinous pericarditis occurring 2-6
depressions on the anterior wall leads). weeks post-MI).

MEHLMANMEDICAL.COM 18 MEHLMANMEDICAL.COM 19
MEHLMANMEDICAL.COM MEHLMANMEDICAL.COM

- ECG is first step in Dx of pericarditis, but USMLE wants echocardiography as


next best step in order to visualize a concomitant effusion that can occur
sometimes. Vignette will give you stereotypical pericarditis + will ask for next
best step in diagnosis; ECG might not be listed and you’re like huh? à Answer is
echocardiography to look for potential effusion concomitant to the pericarditis.
- I should make note that chronic constrictive pericarditis is a separate condition
that doesn’t present with the standard pericarditis findings as described above.
- This is low-yield for USMLE, but students ask about it because it can be
confused with tamponade.
- There’s two ways this can show up:
1) Tuberculosis is a classic cause; there may or may not be calcification
around the heart on imaging. So if you get a Q where patient has TB +
some sort of heart-filling impairment à answer = chronic constrictive
pericarditis. - You can see the amplitudes (i.e., heights) of the complexes are short. This
2) Kussmaul sign will be seen in the Q, where JVD occurs with refers to “low-voltage.” You can also see the heights ever so slightly oscillate up
inspiration rather than expiration. and down. This refers to electrical alternans. They show this ECG twice on 2CK
- Normally, inspiration facilitates RA filling (¯ intrathoracic pressure à ­ NBMEs.
Chronic constrictive pulmonary vascular compliance/stretching à ­ high-low pressure gradient - If the Q asks for next best step in diagnosis, choose ECG as first step if listed If
pericarditis from right heart to the lungs à ¯ in afterload on RV from the lungs à blood not listed, then choose echocardiography, which confirms fluid over the heart.
moves easier from right heart to the lungs à blood is pulled easier from - If the Q asks for next best step in management for tamponade when the
SVC/IVC to the RA). vignette is obvious, choose pericardiocentesis or pericardial window. USMLE
- However, if there is ­ compressive force on the heart, the ­ in negative will not list both; it will be one or the other. NBME 8 offline for 2CK has
intrathoracic pressure during inspiration is not transmitted to the right side of pericardial window as answer, where pericardiocentesis isn’t listed.
the heart, so JVP does not ¯ (and can even paradoxically can ­). - HY type of VT that has sinusoidal pattern on ECG.
- In tamponade, however, as discussed below, the ­ in negative intrathoracic
pressure during inspiration is able to be transmitted to the right side of the
heart, so Kussmaul sign does not occur. This is likely because in constrictive
pericarditis, the rigid pericardium prevents expansion of the right heart
altogether, whereas in tamponade, the pericardium isn’t rigid per se, but is just
filled with blood that can move/shift during the respiratory cycle, thereby
allowing right heart expansion during inspiration.
- Cardiac tamponade = pericardial effusion + low blood pressure. Torsades de pointes
- What determines whether we have a tamponade or not is the rate of (TdP) - USMLE wants you to know this can be caused by some anti-arrhythmic agents,
accumulation of the fluid, not the volume of the fluid – i.e., a stab wound or such as the sodium- and potassium-channel blockers, such as quinidine and
post-MI LV free-wall rupture resulting in fast blood accumulation, even if ibutilide, respectively. They ask this directly on the NBME exam, where Q will
smaller volume, might cause tamponade, but cancer resulting in slow, but large, say patient is given ibutilide + what is he now at increased risk of à answer =
accumulation might not cause tamponade. torsades.
- Tamponade presents as Beck triad: 1) hypotension, 2) JVD, 3) muffled/distant - QT prolongation is risk factor for development of TdP. Agents such as anti-
heart sounds. The question will basically always give hypotension and JVD. psychotics, macrolides, and metoclopramide prolong the QT.
Pericardial effusion / - Tx USMLE wants is magnesium (asked directly on new 2CK form), which
Occasionally they might not mention the heart sounds. But you need to
Cardiac tamponade stabilizes the myocardium in TdP.
memorize Beck triad as HY for tamponade.
- Pulsus paradoxus (i.e., drop in systolic BP >10 mm Hg with inspiration) is - Seen in hyperkalemia.
classically associated with tamponade, although not frequently mentioned in
vignettes. I’ve seen a 2CK NBME Q where they say “the pulsus paradoxus is <10
mm Hg,” which is their way of saying the Dx is not tamponade. I consider that
wording odd, but it’s what the vignette says.
- ECG will show electrical alternans / low-voltage QRS complexes.
Peaked T wave

- Asked once on one of the 2CK forms, where they show the ECG.
- Highest yield point is that if a patient has hyperkalemia and ECG changes, the
Tx USMLE wants is IV calcium gluconate or calcium chloride, which stabilizes the

MEHLMANMEDICAL.COM 20 MEHLMANMEDICAL.COM 21
MEHLMANMEDICAL.COM MEHLMANMEDICAL.COM

myocardium. Calcium gluconate is classic, but calcium chloride shows up as an


answer on a 2CK NBME.
- Means hypokalemia.

- Don’t treat on USMLE.


- No gradual prolongation of PR interval, followed by a random dropping
of the QRS.
U-wave

- Shows up on NBME 12 for 2CK in anorexia patient. First time I’ve ever seen it
show up anywhere on NBME material. But Q doesn’t ride on you knowing it
means hypokalemia to get it right. It’s HY and pass-level to know that purging
(anorexia or bulimia) causes hypokalemia anyway. - Can also sometimes occur as patterns of 2:1, 3:1, etc., where there will
- Seen in Wolff-Parkinson-White syndrome (WPW; accessory conduction be a P to QRS ratio of 2:1 or 3:1, etc.
pathway in heart that bypasses the AV node, resulting in reentrant SVT).
- Classically described as a “slurred upstroke” of the QRS, where the PR interval Second degree Mobitz type II
is shortened.

- Regardless as to whether the dropped QRS is random or in a numerical


Delta wave pattern, there is no gradual prolongation of the QRS before the dropped
complex.
- More dangerous than Mobitz I. This is because Mobitz II has higher
chance of progression into type III heart block.
- Treatment on USMLE is insertion of pacemaker. This is asked on a new
2CK NBME exam.
- Two things you want to look for on ECG:
- Both the delta-wave and WPW have basically nonexistent yieldness on USMLE, - 1) Ultra-slow HR (i.e., 30-40). You’ll see the QRS’s are super far apart.
but I mention them here so you are minimally aware. This is the ventricular escape rhythm.
- They mean hypothermia. You don’t need to be able to identify on ECG. Just - 2) No relationship between the P-waves and QRS complexes.
J waves know they exist, as they show up in a 2CK vignette where patient has body
temperature of 89.6 F (not 98.6).

Third degree

HY Heart block points for Surg


- Prolonged PR interval (>200 ms). Should normally be 80-120 ms.

- Treatment on USMLE is insertion of pacemaker.


- So what you want to remember is that Mobitz II and 3rd-degree are the
ones where we insert pacemaker; 1st-degree and Mobitz I we don’t.

First degree

HY Cardiomyopathy points for USMLE


- Note that above on the ECG, the PR-segment in particular (just prior to
- Can be isolated ventricular or diffuse 4-chamber dilation. Causes are
the QRS complex) is extra-long.
multifarious, but a key feature is systolic dysfunction, where ejection
- Not really assessed on USMLE. Just know the definition.
Dilated (DCM) fraction is reduced (i.e., <55%, where normal range is 55-70).
- Don’t treat on USMLE.
- CXR shows enlarged cardiac silhouette.
Second degree Mobitz type I - Gradually prolonging PR interval until QRS drops. Then cycle repeats.
(aka Wenckebach)

MEHLMANMEDICAL.COM 22 MEHLMANMEDICAL.COM 23
MEHLMANMEDICAL.COM MEHLMANMEDICAL.COM

- Caused by mutations in b-myosin heavy-chain gene; autosomal


dominant; results in disordered/disarrayed myocardial fibers.
- HOCM causes asymmetric septal hypertrophy that results in the
anterior mitral valve leaflet obstructing the LV outflow tract (so it can
sound similar to aortic stenosis).
- Classically sudden death in young athlete; cause of death is ventricular
fibrillation due to acute left heart strain in the setting of fast heart rate.
- Presents with systolic murmur that worsens with Valsalva or standing
(aortic stenosis, in contrast, gets softer or experiences no change with
Valsalva) or standing.
- HOCM and MVP are the only two murmurs that get worse with less
volume in the heart. All other murmurs get worse with more volume.
Hypertrophic obstructive
Valsalva increases intra-thoracic pressure and decreases venous return,
(HOCM)
so there’s less volume in the heart. Standing simply decreases venous
return.
- NBME wants you to know that ­ HR from exercise or stress means
“diastole is shortened more than systole.”
- Beta-blockers (metoprolol or propranolol are both answers on NBME)
are given to slow heart rate, which maximizes diastolic filling and
decreasing symptoms / risk of death.
- An S3 heart sound can sometimes be heard. - Implantable cardioverter-defibrillator (ICD) is indicated if patient
- Cardiac exam shows lateralized apex beat. Can be described as the develops any type of arrythmia, syncope, low blood pressure, or LV wall
point of maximal impulse being in the anterior axillary line. Should be thickness >30mm.
noted that this lateralization just means an enlarged LV, so it is non- - Septal myectomy is done if the left ventricular outflow tract pressure
specific, and can also be seen in LV hypertrophy from any cause. But gradient is >50mm Hg. But it should be noted ICD is usually first-line.
many vignettes will mention it. - Heart failure due to diastolic dysfunction, where HTN is not the cause.
- The arrows USMLE wants are: ¯ EF; ­ LVEDV; ­ LVEDP. I discuss this - JVD is HY for RCM. An S4 can also be seen. The heart will not be dilated.
stuff in more detail in my HY Arrows PDF. - HY causes are Hx of radiation (leads to fibrosis), amyloidosis, and
- Causes of DCM are ABCD: hemochromatosis.
- A: Alcohol. - Student might say, “I thought you said hemochromatosis was DCM. So
- B: Wet Beriberi (thiamine deficiency). if we have to choose on the exam, which one is it?” The answer is,
- C: Coxsackie B virus, Cocaine, Chagas disease. whichever the vignette gives you. If they say a large cardiac silhouette
- D: Drugs à doxorubicin (aka Adriamycin). Restrictive (RCM)
with an S3 and lateralized apex beat, that’s DCM. If they say JVD + S4 +
- Other notable causes are: nothing about a lateralized apex beat, you know it’s RCM.
- Pregnancy (peripartum cardiomyopathy); hemochromatosis; rheumatic - Amyloidosis is protein depositing where it shouldn’t be depositing.
heart disease (myocarditis leading to DCM). Highest yield cause of amyloidosis on USMLE is multiple myeloma, which
- Heart failure due to systemic hypertension. will lead to RCM.
- Characterized by diastolic dysfunction (the heart can pump just fine but - Since RCM is diastolic dysfunction, the arrows are the same as HCM,
cannot expand as easily). which are: « EF; « LVEDV; ­ LVEDP.
- Can be associated with S4 heart sound.
- The arrows USMLE wants are: « EF; « LVEDV; ­ LVEDP.
- Ejection fraction is normal because the heart can pump perfectly fine.
- Students get confused about LVEDV, thinking it should be low, if the HY Angina points for USMLE
heart cannot expand as easily. But this is not the case for USMLE. They - Chest pain that occurs predictably with exercise.
want you to know normal volume can be achieved; it just merely requires - Due to atherosclerotic plaques causing >70% occlusion; can be calcific.
Hypertrophic (HCM) more force/pressure to get there. - Classically causes ST depressions on ECG.
- As with DCM, the apex beat / point of maximal impulse can be - Nitrates (e.g., sublingual isosorbide dinitrate) used as Tx à nitrates “donate”
lateralized, which merely reflects LV hypertrophy. nitric oxide (NO) that upregulates guanylyl cyclase within venous smooth
- As described earlier, paradoxical splitting of S2, left-axis deviation on muscle à increased cGMP à relaxation of venous smooth muscle à increased
ECG, and LBBB can all be seen due to LVH. When you see these findings Stable angina
venous pooling of blood à decreased venous return à decreased myocardial
in vignettes, don’t get confused. They just mean LVH. oxygen demand à mitigation of chest pain.
- Often associated with hypertensive retinopathy (fundoscopy shows - Nitrates are contraindicated with PDE-5 inhibitors (e.g., Viagra) due to risk of
narrowing of retinal vessels, flame hemorrhages, and “AV-nicking”), low blood pressure.
hypertensive nephropathy (hyperplastic arteriolosclerosis with increased - Sodium nitroprusside used for hypertensive emergencies dilates arterioles in
creatinine). addition to the veins. If USMLE asks you where this drug acts, choose arterioles.
Unstable angina - Chest pain that is unpredictable and can occur at rest.

MEHLMANMEDICAL.COM 24 MEHLMANMEDICAL.COM 25
MEHLMANMEDICAL.COM MEHLMANMEDICAL.COM

- Due to partial rupture of atherosclerotic plaque leading to partial occlusion. - Janeway lesions, Osler nodes, splinter hemorrhages, etc., are low-yield for
- ST depressions on ECG. USMLE and mainly just school of medicine talking points.
- Diltiazem is answer on new 2CK NBME for patient with unstable angina. - HACEK organisms nonexistent on USMLE.
- Patients need cardiac catheterization. - Blood cultures before antibiotics is important for 2CK.
- Vasospastic angina that occurs at rest (i.e., watching TV or while sleeping) in - Transesophageal echocardiography (TEE) confirms diagnosis after blood
younger adults; it is not caused by atherosclerosis. cultures. Transthoracic echocardiography (TTE) is not done for endocarditis.
- ST elevations are seen on ECG. - For 2CK, empiric treatment for endocarditis is vancomycin, PLUS either
- You must know that Prinzmetal is also known as variant angina pectoris. There gentamicin or ampicillin/sulbactam.
Prinzmetal angina is an NBME Q that gives vignette of Prinzmetal, but answer is “variant angina - Vancomycin targets gram-positives (including MRSA). Gentamicin targets
(variant angina pectoris) pectoris.” gram-negatives.
- Treatment is nitrates (can cause coronary artery dilation unrelated to the - Endocarditis prophylaxis given prior to a dental procedure is usually ampicillin
venous pooling effects) or dihydropyridine calcium channel blockers (e.g., or a second-generation cephalosporin, such as cefoxitin.
nifedipine). Avoid a1-agonists in these patients (cause vasoconstriction), as well Management - Indications for endocarditis prophylaxis are:
as non-selective b-blockers like propranolol (can cause unopposed a effects). 1) Hx of endocarditis (obvious);
2) If there is any prosthetic material in the heart whatsoever;
3) If there is any congenital cyanotic heart disease that has not been completely
repaired (if it’s been completely repaired with prosthetics, give prophylaxis);
Hypertensive Emergency + urgency 4) Hx of heart transplant with valvular regurgitation of any kind.
- HTN >180/120 + signs of end-organ damage. - Highest yield point for USMLE about endocarditis prophylaxis is that mitral
- The latter can be hypertensive encephalopathy (confusion), nephropathy (poor renal valve prolapse (MVP) and valve regurgitations or stenoses are not an indication.
function tests), retinopathy, acute heart failure, etc. In other words, do not give prophylaxis if the patient has MVP, MR, AS, etc. In
Emergency - BP should be ¯ by no more than 20-25% in the first hour, as drastic ¯ can compromise addition, bicuspid aortic valve is not an indication.
perfusion to the brain and vital organs.
- Blood pressure should be brought under 160/100 by 24-48 hours.
- Drugs used are IV sodium nitroprusside, IV nicardipine, IV labetalol, and oral captopril.
- HTN >180/120 + no signs of end-organ damage. Conditions confused for cardiac path
Urgency - Blood pressure should be brought under 160/100 by 24-48 hours. - Psych forms love trying to make you think this is an MI.
- Drugs used are IV sodium nitroprusside, IV nicardipine, IV labetalol, and oral captopril. - They’ll give you young, healthy patient who feels doom / like he or she is
going to die.
- Sometimes they mention in stem Hx of MI in family as distraction.
Panic attack - They can say patient has mid-systolic click, as discussed earlier, and then
HY Endocarditis points they ask for cause of patient’s symptoms à answer = panic disorder, not
- Bacterial infection of valve in patient with no previous heart valve problem. MVP. Student gets confused, but MVP is almost always asymptomatic, where
- Caused by Staph aureus on USMLE. panic attack is clearly cause of the patient effusively hyperventilating.
- Left-sided valves (i.e., aortic and mitral) most commonly affected because of - Treat with benzo.
greater pressure changes (i.e., from high to low) within left heart, resulting in - Orthostatic hypotension is defined as intravascular fluid depletion causing a
Acute endocarditis drop of systolic BP >20 mmHg and diastolic BP >10 mmHg when going from
turbulence that enables seeding.
- IV drug users à venous blood inoculated with S. aureus à travels to heart and supine to standing.
causes vegetation of tricuspid valve. Orthostasis - Shows up on 2CK IM form as exactly a drop of 20 and 10, respectively, for
- Staph aureus is coagulase positive. systolic and diastolic BPs in a patient with fainting à answer = “intravascular
- Bacterial infection of valve in patient with history of valve abnormality (i.e., fluid depletion.”
congenital bicuspid aortic valve, Hx of rheumatic heart disease). - Diuretic use is big risk factor.
- Caused by Strep viridans on USMLE. You need to know S. viridans is can be - Fainting in response to stressor (e.g., emotional trigger).
Subacute endocarditis - Stress triggers an initial sympathetic response, which in turn triggers a
further broken down into: S. sanguinis, S. mutans, and S. mitis.
- Hx of dental procedure is HY precipitating event, where inoculation of blood compensatory parasympathetic response. This latter response is excessive in
occurs via oral cavity à previously abnormal valve gets seeded. Vasovagal syncope some people, where the peripheral arterioles dilate and the heart slows too
- New-onset murmur + fever = endocarditis till proven otherwise on USMLE. much à decreased cerebral perfusion à lightheadedness/fainting.
- Reactive thrombocytosis (i.e., high platelets) can occur due to infection. This is - 2CK wants you to know a tilt-table test can be used to diagnose, where a
not unique to endocarditis, but it is to my observation USMLE likes endocarditis reproduction of symptoms can occur.
as a notable etiology for it. In other words, if you get an endocarditis question - USMLE likes this for both Steps 1 and 2.
Random points and you’re like, “Why the fuck are platelets 900,000?” (NR 150-450,000), don’t - They’ll say dude was shaving then got lightheadedness or fainted.
Carotid sinus
be confused. Mechanism is ­ stretch of carotid sinus baroreceptors à ­ afferent CN IX
hypersensitivity
- Hematuria can occur from vegetations that launch off to the kidney. firing to solitary nucleus of the medulla à ­ efferent CN X parasympathetic
- Endocarditis + stroke-like episode (i.e., focal neurologic signs) = septic firing down to cardiac nodal tissue à ¯ HR à ¯ CO à ¯ cerebral perfusion.
embolus, where a vegetation has launched off to the brain. Costochondritis - Inflammation of cartilage at rib joints.

MEHLMANMEDICAL.COM 26 MEHLMANMEDICAL.COM 27
MEHLMANMEDICAL.COM MEHLMANMEDICAL.COM

- Will present as chest pain that worsens with palpation or when patient - All patients with arterial disease should be on triad of 1) ACEi/ARB, 2) statin, 3)
reaches over the head or behind the back. These two findings are clear anti-platelet therapy (same as carotid stenosis). This is unrelated to the
indicators we have an MSK condition, not cardiac. management sequence of exercise program à cilostazol à surgery.
- Can be idiopathic, caused by strain (e.g., at the gym), or even post-viral. - Congestion of venous system usually from valvular incompetence; idiopathic /
- MSK condition asked twice on 2CK material that has nothing to do with the familial; varicose veins are one type of venous disease and are not synonymous;
lungs, despite the name. patients can have venous disease without varicosities.
- This is viral infection (Coxsackie B) causing sharp lateral chest pain due to - Peripheral pulses are normal (those reflect arteries, not veins).
Pleurodynia
intercostal muscle spasm. Sometimes students choose pericarditis, etc., even - Lower legs demonstrate “brawny edema,” which is a brown, hemosiderin-
though the presentations are completely disparate. laden edema due to ­ pressure / micro-extravasations; hyperpigmentatory
- Creatine kinase can be elevated in stem due to ­ tone of muscle. changes resulting in brown/red skin is known as stasis dermatitis, aka post-
- Viral infection causing inflammation of the pleura (layers covering the lungs), phlebitic syndrome; the latter is a term is asked on 2CK, so know the annoying
leading to sharp chest pain. vocab.
Viral pleurisy
- If this is the answer, CK will be normal (unlike pleurodynia, because it’s not - Venous ulcers are large and sloughy, and located at the malleoli.
MSK).
- Can cause angina-like pain in patient without cardiovascular disease.
Diffuse esophageal spasm
- I discuss DES more in Gastro chapter.
- Can present as chest pain confused for MI. ECG will be normal, clearly.
Gastroesophageal reflux
- I discuss GERD more in Gastro chapter.

Venous disease
Arterial vs venous disease
- Caused by atherosclerotic disease; presents as diminished peripheral pulses in
patient over 50 who has risk factors, e.g., diabetes, smoking, HTN.
- Lower legs can be shiny and glabrous (trophic changes).
- Arterial ulcers are small and punched-out; located on tops/bottoms of feet and
toes. - Diagnose with venous duplex ultrasonography of the legs; first treatment is
compression stockings. Never choose answers such as venous stripping or glue
agents, etc.
- Venous disease ­ risk for DVT and superficial thrombophlebitis (STP). If patient
has active DVT or STP, answer = subcutaneous enoxaparin (heparin) over
compression stockings.
- Surgery is indicated if there are significant skin changes, venous ulcers, or for
symptomatic varicose veins (i.e., pain, itching, swelling, cramps).
- Sclerotherapy is often chosen as the first surgery, where a sclerosing (scarring)
agent is injected into the vein, causing it to close where the blood is re-routed
to other veins. I mention this for the sake of our Surgery PDF here, but on NBME
Qs, I’ve never seen surgery as correct. The answer is basically always just
Arterial disease compression stockings, or rarely heparin for DVT or STP as I mentioned above.

- Ankle-brachial indices (ABIs) are first step in diagnosis (exceedingly HY on 2CK),


which compare BP in ankle to the arm; if <0.9, this reflects ¯ peripheral blood Cardiac stress test points
flow due to atherosclerosis. - Most 2CK Qs that ask about stress tests are in the context of evaluating patients for perioperative MI risk.
- If ABIs are not listed as first step in diagnosis for whatever reason, choose - It is rare the Q will force you to choose between different types of stress tests. 4/5 Qs will just list one
Doppler ultrasound. There is one 2CK NBME Q where this is the case. stress test, where it is simply assessing, “Do you know a stress test should be done, period, in this scenario.”
- After ABIs, next step is exercise stress test (if listed) in order to determine - Stress tests are also done for peripheral arterial disease prior to recommending an exercise/walking
exercise tolerance. If not listed, go straight to “recommend an exercise / program (as mentioned above).
walking program.” Do not choose cilostazol first or arteriography as answers. - Most common stress test.
- 2CK CMS Surg form 5 has “prescription for an exercise program” as the - The answer on USMLE for patients who have stable angina, where you’re looking
answer. Students say, “Why does it say ‘prescription’?” No fucking idea. Ok? for ST depressions (i.e., evidence of ischemia) with exertion.
- Surgery is indicated in the event of critical limb ischemia, which is when there Exercise ECG - Requires a patient has a normal baseline ECG in order to perform.
is chronic ischemic rest pain, ulcers, or gangrene. - In other words, the Q will give you a big 15-line paragraph + mention in the last line
- First surgical intervention is usually angioplasty +/- stenting. Endarterectomy that the patient’s baseline ECG shows, e.g., a LBBB from a year ago that’s unchanged.
and bypass surgery are indicated for more severe blockages. This means ECG stress test is wrong in this situation, since you need to have a normal

MEHLMANMEDICAL.COM 28 MEHLMANMEDICAL.COM 29
MEHLMANMEDICAL.COM MEHLMANMEDICAL.COM

ECG to do it. The 1/5 Qs that force you to choose between stress tests want you to Vasculitides (pleural for vasculitis) for Surg
know this detail, basically, where you just choose the non-ECG stress test instead. - Medium-vessel vasculitis that causes a “string of pearls” appearance of the
- Used to look for heart failure (i.e., ¯ EF) with exertion, not overt ischemia. renal vessels.
- In other words, the answer on USMLE for patients who don’t get chest pain with
Exercise echo exertion (i.e., don’t have stable angina), but who get shortness of breath with
exertion. This reflects, at a minimum, left heart decompensation with possible ¯ EF.
- Also the answer for patients who have abnormal baseline ECG.
- Refers to numerous answer choices on USMLE – i.e., dobutamine-echo,
dipyridamole-thallium.
- The answer on USMLE for patients who cannot exercise, such as in the setting of
angina when merely walking up a single flight of steps, or in patients imminently
undergoing major surgery (e.g., AAA repair), where perioperative MI risk needs to be
assessed. I have seen both of these scenarios on 2CK forms.
- The USMLE will typically not force you to choose between stress tests. As I Polyarteritis nodosa (PAN)
Pharmacologic
mentioned at the top of this table, they will usually just have the pharmacologic
stress test as the only one listed.
- Dobutamine is a b1-agonist that stimulates the heart (i.e., ­ oxygen demand). Echo
can then be done to look for ¯ EF (i.e., heart failure).
- Dipyridamole is a phosphodiesterase inhibitor that dilates arterioles. HR goes up to
compensate, thereby ­ myocardial oxygen demand. Thallium is then used to look at
perfusion of the myocardium.
- “Cardiac scintigraphy” is a broad term that refers to any evaluation of the heart in - Causes fibrinoid necrosis, which means it looks like fibrin but it ain’t fibrin.
which some form of radiotracer is used (i.e., thallium, technetium, sestamibi). - Offline NBME has “segmental ischemic necrosis” as the answer.
- This is the same as pharmacologic stress test for all intents and purposes on USMLE, - Can be caused by hepatitis B.
Cardiac - For whatever reason, USMLE wants you to know PAN spares the lungs – i.e.,
even though technically it need not require myocardium is stimulated and can just be
scintigraphy
used to look at blood flow to the heart in the resting state. it does not affect the pulmonary vessels.
- The point is: This is an answer on 2CK sometimes as just another way of them - Aka “pulseless disease.” Classically affects Asian women 40s or younger.
writing “pharmacologic stress test.” Choose it if the patient cannot exercise. - Inflammation of large vessels, including the aorta.
Takayasu arteritis
- “Myocardial perfusion scan” is one type of cardiac scintigraphy that evaluates blood - Always affects the subclavian arteries (which supply the arms), which is why
flow to myocardium. It is non-invasive, whereas coronary angiography is invasive and it can cause weakly, or non-palpable, pulse in the upper extremities.
Myocardial
evaluates coronary blood flow via the use of a catheter. - Aka giant cell arteritis.
perfusion scan
- This is interchangeable with cardiac scintigraphy and pharmacologic stress test on - 9/10 Qs will be painful unilateral headache in patient over 50. I’ve seen one
USMLE for all intents and purposes. Q on NBME where it’s bilateral.
- Flares can be associated with low-grade fever and high ESR.
- Patients can get proximal muscle pain and stiffness. This is polymyalgia
rheumatica (PMR). The two do not always go together, but the association is
Knee trauma causing popliteal arterial injury HY. (Do not confuse PMR with polymyositis. The latter will present with ­ CK
- If Q gives you MVA where the knee is injured + absent pulses distally, the sequence of and/or proximal muscle weakness on physical exam. PMR won’t have either of
answers they want is knee relocation first, followed by arteriography to look for these findings. I talk about this stuff in detail my MSK notes.)
popliteal artery injury. - Patients can get pain with chewing. This is jaw claudication (pain with
Knee dislocation chewing).
- In one of the 2CK Qs, they already tell you the knee is relocated, then the answer is
“arteriography with runoff.” Students say, “what’s the runoff part?” No fucking idea, - Highest yield point is we give steroids before biopsy in order to prevent
Temporal arteritis
it’s just what they want. blindness.
- If Q gives gunshot wound to the knee + absent distal pulses, go straight to “surgical - An NBME has “ischemic optic neuropathy” as the answer for what
Penetrating exploration” as the answer. complication we’re trying to prevent by giving steroids in temporal arteritis.
trauma - This could be thought of as the knee-equivalent of a gunshot wound to the abdomen, - IV methylprednisolone is typically the steroid given, since it’s faster than oral
where straight to laparotomy (even if patient is stable) is the answer. prednisone.
- It’s to my observation many 2CK NBME Qs will give the answer as something
like, “Steroids now and then biopsy within 3 days,” or “IV methylprednisolone
and biopsy within a week.” Students ask about the time frames, but for
whatever reason USMLE will give scattered/varied answers like that.
- Another 2CK Neuro CMS Q gives easy vignette of temporal arteritis and then
asks next best step in diagnosis à answer = biopsy. Steroids aren’t part of the
answer. Makes sense, since they’re asking for a diagnostic step.
Thromboangiitis obliterans - Aka Buerger disease; technically a vasculitis.

MEHLMANMEDICAL.COM 30 MEHLMANMEDICAL.COM 31
MEHLMANMEDICAL.COM MEHLMANMEDICAL.COM

- Dry gangrene of the fingers or toes seen generally in male over 30 who’s a - Important for 2CK Obgyn Qs.
heavy smoker. - The answer on USMLE in a woman who has post-partum endometritis (fever
- Treatment is smoking cessation. + tender lower abdomen) with persistent fever >48 hours despite antibiotics.”
- Don’t confuse with Berger disease, which is IgA nephropathy. Pelvic septic - Endometritis can lead to ­ risk of local infective clots in the ovarian veins.
- Tertiary syphilis can cause ascending aortitis + aortic aneurysm. thrombophlebitis (PST) - If they give you a post-partum woman with sepsis (i.e., SIRS + infection), but
Ascending aortitis
- Causes “tree-barking” of the aorta. the vignette doesn’t fit PST as described above, the answer is “puerperal
sepsis” on the Obgyn form. The latter is a more general term and can refer to
many causes of post-partum sepsis (including PST confusingly enough).
- Painful palpable cord in the ankle that may or may not track up to the knee.
Thrombophlebitis - Seen in patients with venous insufficiency.
- DVT will be unilateral thigh or lower leg swelling in patient with risk factors - Answer is “subcutaneous enoxaparin.” Compression stockings are typically
Superficial
such as: post-surgery, prolonged sedentation, OCP use, Hx of thrombotic the answer for first step in venous insufficiency, but if you have an active ST or
thrombophlebitis
disorders (e.g., Factor V Leiden, prothrombin mutation). DVT, heparin must be given as first step.
- Virchow triad for ­ DVT risk: 1) venous stasis (e.g., post-surgery sedentation), - There will occasionally be some intentional redundancy on my end with
2) hypercoagulable state (e.g., estrogen use, underlying malignancy), 3) things I write in this doc if I believe they’re HY enough (as with this).
endothelial damage (i.e., smoking).
- OCPs contraindicated in smokers over 35 because estrogen causes
hypercoagulable state for two reasons: 1) estrogen upregulates fibrinogen; 2)
estrogen upregulates factors Va and VIIIa. Random HY cardio pharm points
- USMLE loves nephrotic syndrome as cause of DVT (loss of antithrombin III in - Non-dihydropyridine calcium channel blocker (acts on nodal calcium channels).
the urine à hypercoagulable state). - Causes constipation.
Verapamil
- Antiphospholipid syndrome à DVTs despite paradoxical ­ PTT (i.e., if PTT is - Used for AF for rate control sometimes in place of metoprolol (don’t worry about
high, you’d think you have bleeding diathesis, not thromboses); may or may the use-case; USMLE doesn’t give a fuck; just know MOA and side-effect).
not be due to SLE. Antibodies against phospholipids cause in vivo clumping of - Dihydropyridine calcium channel blocker (acts on arteriolar calcium channels à
platelets + ­ clot initiation, but disruption of in vitro PTT assay means ­ PTT. dilates arterioles à ¯ peripheral vascular resistance à ¯ BP).
- Major danger is DVT can embolize to lungs causing PE à acute-onset - Causes peripheral edema / fluid retention.
Nifedipine
shortness of breath and tachycardia + death if saddle embolus. - Used for essential HTN in patients without diabetes, atherosclerotic disease, or
Deep vein thrombosis - Homan sign can mean DVT, which is pain in the calf with dorsiflexion of foot. renal disease (if that sounds confusing, I talk about this in extensive detail in my HY
- Diagnose DVT with duplex venous ultrasound of the leg/calf. Risk Factors PDF).
- Treatment is heparin. - Can cause depression and sexual dysfunction.
- Harder surgery stuff for 2CK is that they care about prophylactic vs - Avoid in patients with lung disease or history of severe or psychotic depression. No
therapeutic doses of heparin. Prophylactic dose is lower-dose and is used longer contraindicated in peripheral vascular disease in most cases.
perioperatively in patients with venous disease/stasis or who are high risk. If a - Metoprolol used for rate-control in AF.
b-blockers
patient has an actual full-blown DVT, however, give therapeutic dose, which is - Labetalol used first-line in patients who have aortic dissection and traumatic
higher-dose. rupture of the aorta. Nitroprusside comes after.
- There are two 2CK Qs on this stuff. One just mentions a guy going into - 2CK Q gives “esmolol + nitroprusside” as answer to a traumatic rupture Q, but
surgery who has Hx of venous stasis à answer = “prophylactic heparin dose”; almost always, they will just want “labetalol.”
“therapeutic heparin dose” is wrong answer. - Potassium channel blocker.
- The second question gives a guy who’s already on prophylactic heparin but Amiodarone - Can cause TdP, greyish skin discoloration, and thyroiditis.
gets a DVT anyway. The answer is then “heparin.” It’s weird because students - Used for VT in patients without coma or low BP.
are like, “Wait what? He’s already on heparin though.” And I’m like, yeah, but - Sodium channel blocker.
what they mean is, we have to give therapeutic dose now for the active DVT, Quinidine
- Can cause TdP and cinchonism (headache + tinnitus).
which is higher dose. - Directly blocks myocardial Na+/K+ ATPase pump à causes indirect inactivation of
- DVT can rarely cause stroke if an ASD is present (paradoxical embolus). myocardial Na+/Ca2+ ATPase à more Ca2+ remains in myocardial cell à increased
Dumb and low-yield, but it shows up, and students get fanatical over it. contractility.
- Thrombophlebitis means inflammation of a vein. - Also has parasympathomimetic effect at nodal tissue that slows HR.
- Post-surgery, this is usually due to changes in hemostasis and coagulability. Digoxin
Post-op migratory - In other words, digoxin both slows HR + increases contractility.
- Will present as pink/red painful lesions appearing asymmetrically on the - Hypokalemia can cause toxicity. This is because digoxin binds to extracellular K+
limbs within days of surgery. You just need to be able to diagnose this. binding site, so if less K+ is around to compete, lower dose is needed to induce effect.
- Migratory thrombophlebitis classically due to head of pancreas - Toxicity presents classically as yellow/wavy “Vincent van Gogh” vision.
Trousseau sign of
adenocarcinoma. But this can also be seen with adenocarcinomas in general, - USMLE-favorite ACE inhibitor.
malignancy
e.g., pulmonary. - Can cause dry cough; also can ­ serum K+; avoid in hereditary angioedema.
- Shows up on 2CK Surg form as patient who had a catheter in and then Lisinopril
- Used for HTN in patients with pre-diabetes, diabetes, atherosclerotic disease, or
Catheter-associated septic develops a 4-cm indurated, painful, fluctuant cord in his arm (refers to vein). renal disease (I talk about this in HY Risk Factors PDF in more detail).
thrombophlebitis (CAST) - Answer = “excision of vein.” Obscure question, but not my opinion. Take it Valsartan - Angiotensin II receptor blocker (ARB).
up with NBME if you think it’s weird.

MEHLMANMEDICAL.COM 32 MEHLMANMEDICAL.COM 33
MEHLMANMEDICAL.COM MEHLMANMEDICAL.COM

- Use-cases are identical on USMLE to ACEi (i.e., if you see both as answer choices to - Can cause drug-induced lupus with anti-histone antibodies.
a question, they’re usually both wrong because they’re the “same”). - Sodium channel blocker used as first-line rhythm control in patients fail rate-control
- Doesn’t cause dry cough the way ACEi do. for AF.
Flecainide
Isoproterenol - b1/2 agonist à increases HR and decreases peripheral vascular resistance. - Patient should have no structural or coronary artery disease. Otherwise use a
- a1 agonists à constrict arterioles à ­ BP à HR ¯ due to baroreceptor reflex. potassium channel blocker like amiodarone or dofetilide.
- Highest yield uses on USMLE are for nasal decongestion à constrict capillaries Dofetilide, - Potassium channel blockers.
within nasal mucosa à ¯ inflammation à relief of congestion. Ibutilide - Can cause torsades (asked directly on NBME).
- Can cause rhinitis medicamentosa, which means rebound nasal congestion upon Omega/fish oils - Decrease triglycerides.
Oxymetazoline,
withdrawal if used non-stop for ~5 days. In other words, patients should use only as - 2CK Surgery form gives patient who is in pain from surgery despite being on max
Phenylephrine
needed for a maximum of about ~3-4 days while sick. doses of morphine + they say blood pressure is elevated + ask what is next best step
- Can theoretically be used in the acute setting of nasal bleeding. For surgery, choose à answer = “increase bolus of morphine.” Unusual, since they say max dose of
nasal packing first for management. Then answers like a1 agonists and cauterization morphine already, but 1) we always full treat pain, and 2) explanation for the Q talks
can be considered. about how pain can lead to high BP.
- Dilates arterioles à ¯ BP. - My point here is that just be aware HTN can be caused by pain à treating pain can
Hydralazine - Used for hypertensive emergencies in pregnancy. bring down the BP.
Morphine
- Affects calcium currents (but not a calcium channel blocker). - Separately, treatment of MI, acute limb ischemia, and sickle crisis usually includes
- As discussed earlier, they liberate NO which ­ guanylyl cyclase à relaxation of oxygen + morphine.
venous smooth muscle à venous dilation/pooling à ¯ preload on heart à ¯ oxygen - Answer on NBME exam for how a patient using a self-administering pump develops
Nitrates morphine overdose = “morphine is converted into active metabolites that
demand à relief of anginal pain.
- For sodium nitroprusside, choose arterioles as site of action. accumulate.”
- Statins have 2 HY MOAs on USMLE: 1) inhibit HMG-CoA reductase; 2) upregulate - Otherwise morphine is odd drug for me to put in this table; didn’t know where else
LDL receptors on hepatocytes. to put it though.
- Can cause myopathy and toxic hepatitis. An offline NBME has myopathy as correct
over toxic hepatitis.
- Indications for statins on 2CK vary depending on the source (i.e., whether to use the
70 vs 100 mg/dL cutoff in certain scenarios), but for USMLE, give if:
- Age 20-39 if LDL > 190 mg/dL.
Statins - Age 40-75 if LDL > 100 mg/dL.
- Age 20-75 in diabetic if LDL >100 mg/dL.
- Some sources use 70, rather than 100, for the latter two cutoffs. What I
can say is that I’ve seen 2CK NBME Qs where they use 100 as the cutoff (i.e.,
they give an LDL of 95 and statin is wrong, implying LDL is satisfactory).
- Some sources incorporate a CVD risk % of >7.5%, which is more of a moot /
pedantic talking point. I’ve seen one 2CK NBME Q where a CVD risk % shows
up in the stem, but it doesn’t rely on you knowing that point to get it right.
- Fibrates (e.g., fenofibrate) upregulate PPAR-a and lipoprotein lipase; best drugs to
decrease triglycerides.
- Give if triglycerides >300 mg/dL.
Fibrates
- Can cause myopathy and hepatotoxicity, as well as cholesterol gall stones.
- If the USMLE asks you why statins + fibrates combined have ­ chance of myopathy,
the answer is “P-450 interaction.”
Ezetimibe - Blocks cholesterol absorption in the small bowel.
- Bile acid sequestrant. Causes reduced enterohepatic circulation of bile acids at
Cholestyramine terminal ileum à liver must now convert more cholesterol into bile acids in order to
replenish them à liver pulls cholesterol out of the blood to accomplish this.
Orlistat - Pancreatic lipase inhibitor used for obesity; can cause steatorrhea.
- Vitamin B3. Two MOAs USMLE wants you to know: 1) ¯ VLDL export by the liver; 2)
­ HDL more than any other medication.
- Deficiency à pellagra à 3Ds à dementia, dermatitis, diarrhea.
- Can present as delirium instead of overt dementia (biggest risk factor for delirium is
Niacin underlying dementia, so old patient with delirium often has underlying cognitive
decline); dermatitis will present on USMLE as either hyperpigmentation of the skin of
the forearms or Casal necklace.
- Administration can cause flushing (caused by prostaglandin, so mitigated by asprin),
gout, and insulin resistance.
Procainamide - Sodium channel blocker that can be used in Wolff-Parkinson-White.

MEHLMANMEDICAL.COM 34 MEHLMANMEDICAL.COM 35
MEHLMANMEDICAL.COM MEHLMANMEDICAL.COM

Peds Cardio

HY Valvular / flow abnormalities for Peds


- There’s a lot of detail regarding the following conditions I could discuss at length, but I’m going to keep the
focus on what is sufficient for the pediatrics shelf here (and Steps 2CK/3), which is the point.
- Fixed splitting of S2
- Can sometimes be associated with a systolic flow murmur, since more
blood L à R from the LA à RA means more blood flow across the
pulmonic valve. So Q might say “fixed splitting of S2 and a systolic
murmur.”
- Sometimes can be seen in Qs as “wide, fixed splitting.” I only mention
this because some students get pedantic / ask about this. “Wide splitting”
just means right ventricular hypertrophy. So if the Q says “wide, fixed
splitting,” they’re saying the patient has RVH due to an ASD. - NBME loves this style of Q. You can see O2 somehow increased from RA
- Patent foramen ovale = ASD on USMLE. Don’t confuse with patent ductus to RV. The only way this is possible is if we have a VSD where oxygenated
arteriosus (discussed below). blood moves from LV à RV.
- USMLE loves asking questions showing you change in oxygen in the
chambers of the heart and making you choose ASD, VSD, etc.
Atrial septal defect

- This one might initially appear a little more difficult. This is Eisenmenger
- For example, you can see above that somehow O2 increases from the syndrome, where we have a reversal of flow from RV à LV across the VSD.
SVC to the RA, which is ordinarily impossible. The only way this could occur The NBME is known to show this diagram as well.
is if an ASD is present, where oxygenated blood moved from LA à RA. - Seen in Down syndrome.
- Holosystolic (aka pan-systolic) murmur at lower left sternal border. Atrioventricular septal defect - Between the atrium and ventricle, aka “endocardial cushion defect,”
- Can be associated with a diastolic rumble or enlarged left atrium (if more although this latter term can also apply to ASD and VSD in Downs.
blood going L à R across VSD, then more blood is returning to the LA from - Holosystolic (pan-systolic) or just regular “systolic,” 29 times out of 30.
the lungs à LA dilatation). - Q on NBME 20 offline for Step 1 has MR as “mid-systolic,” but I contend
- Seen as part of tetralogy of Fallot (VSD, RVH, overriding aorta, pulmonic this is erratum.
stenosis). - Most USMLE questions will not mention it radiating to the axilla.
Mitral regurgitation
- If a VSD is repaired, USMLE wants ­ LV pressure, ¯ RV pressure, and ¯ LA - Seen acutely in rheumatic heart disease as a kid (valve scars over years
pressure as the changes now seen in the heart. later and becomes mitral stenosis).
- VSD does not cause cyanosis at birth. Only years later after the higher - Can cause JVD (i.e., back up all the way to the right heart); this is
blood flow to the lungs results in pulmonary hypertension, followed by mentioned numerous times on the new NBMEs.
right ventricular hypertrophy and reversal R à L (Eisenmenger) does the - Described as “rumbling diastolic murmur with an opening snap”; can also
Ventricular septal defect patient become cyanotic. be described as “decrescendo mid-late diastolic murmur” (i.e., following
- Murmur can be silent or soft at birth, followed by loud at 7 days of life. the opening snap).
The USMLE will ask why the murmur is louder now à answer = decreased - Can cause a right-sided S4 if the pressure backs up all the way to the right
pulmonary vascular resistance – i.e., the lungs open up during the first heart (seen on NBMEs sometimes; this confuses students because they
week of life, resulting in decreased RV pressure and an increase in the L à think S4 must be LV, but it’s not the case). An S4 is a diastolic sound heard
R pressure gradient (louder murmur). Mitral stenosis in either the LV or RV when there is diastolic stiffening due to high
- Conversely, if they ask why the murmur was softer at birth compared to afterload.
now, the answer is “increased pulmonary vascular resistance,” where the - 99% of mitral stenoses are due to Hx of rheumatic heart disease (i.e., the
lungs were still closed at the time, so there was a lesser gradient L à R patient had rheumatic fever as a child, where at the time it was mitral
(softer murmur). regurg, but years later it has now become mitral stenosis).
- Similar to ASD Qs, USMLE loves giving you diagrams with changes in O2 - One 2CK NBME Q mentions patient with history of rheumatic heart
between the chambers and then making you infer we have a VSD. disease who, years later, now has 4/6 rumbling diastolic murmur without

MEHLMANMEDICAL.COM 4 MEHLMANMEDICAL.COM 5
MEHLMANMEDICAL.COM MEHLMANMEDICAL.COM

an opening snap; this is still mitral stenosis. Although opening snap is - The bicuspid valve need not calcify in middle-age prior to the AS forming.
buzzy for MS, just be aware it’s not mandatory and that this Q exists on Bicuspid valve can present with AS murmur in child or high schooler.
NBME. - Do aortic valve replacement on 2CK if 1) cross-section of valve is <1.0
- Most common murmur. cm2, or 2) there is SAD. They ask both of these as separate Qs where they
- Described as mid-systolic click. want valve replacement.
- “Myxomatous degeneration” is buzzy term that refers to connective - “Supravalvular aortic stenosis” can occur in Williams syndrome (rare AD
tissue degeneration causing MVP in Marfan and Ehlers-Danlos. disorder with elfin-like facies).
- Almost always asymptomatic. On 2CK forms, they want you to know - Will be described on USMLE as a holosystolic murmur that increases with
about “mitral valve prolapse syndrome,” which is symptomatic MVP that inspiration.
presents as repeated episodes of “fleeting chest pain” on the left side in an - Right-sided heart murmurs get worse with inspiration à diaphragm
otherwise healthy patient 20s-30s, sometimes starting in teenage years. moves down à decreased intra-thoracic pressure à increased right-heart
Mitral valve prolapse They might say there is Hx of MI in the family, but this is MVPS, not MI. filling.
Answer on 2CK form is “no treatment necessary." - Can cause pulsatile liver.
- USMLE loves using MVP as a distractor in panic disorder questions, - Highest yield cause of TR on USMLE is pulmonary hypertension / cor
particularly on the 2CK Pysch CMS forms. They will give long paragraph pulmonale. I see this all over the NBME exams. For whatever reason, these
Tricuspid regurgitation
about panic attack/disorder + also mention there’s a mid-systolic click; conditions do not cause pulmonic regurg; they cause tricuspid regurg. In
they’ll ask for cause of patient’s presentation à answer = panic disorder, other words, if you see tricuspid regurg in a Q, your first thought should be
not MVP à student is confused because they say mid-systolic click, but pulmonary hypertension or cor pulmonale (right heart failure due to a
the MVP isn’t the cause of the patient’s presentation; the panic disorder is; pulmonary cause).
MVP’s are usually incidental, benign, and asymptomatic. - Bronchopulmonary dysplasia can cause cor pulmonale in infants on
- Decrescendo holo-diastolic (pan-diastolic) murmur; can also be described NBME. This is when the neonate is exposed to too much oxygen at birth,
as “early diastolic murmur,” or “diastolic murmur loudest after S2.” usually while in ICU and/or on home oxygen, followed by development of
- Causes wide pulse pressure (i.e., big difference between systolic and fibrotic lung disease à backs up to right heart.
diastolic pressures, e.g., 160/50, or 120/40) à results in head-bobbing and - Nonexistent murmur on USMLE. I don’t think I’ve ever seen this assessed
bounding pulses (don’t confuse with slow-rising pulses of aortic stenosis). once on any NBME exam for Steps 1 and 2 combined.
- The bounding pulses can be described on NBME as “brisk upstroke with Tricuspid stenosis - In theory, would be a rumbling diastolic murmur similar to mitral
precipitous downstroke.” In turn, they can just simply say, “the pulses are stenosis, but would increase with inspiration since it’s on the right side of
brisk,” meaning the systolic component is strong. the heart.
- I would say 4/5 times bounding pulses means AR. The other 1/5 will be - Same as with tricuspid stenosis, this is a nonexistent murmur on USMLE.
PDA and AV fistulae (discussed below). Bounding pulses occur when blood I’ve never seen it assessed.
Pulmonic regurgitation
quickly leaves the arterial circulation. In AR, the blood quickly collapses out In theory it would be the same as aortic regurg but on the right (i.e., holo-
Aortic regurgitation of the aorta back into the LV. In PDA, it leaves the aorta and enters the diastolic murmur), but increases with inspiration.
ductus arteriosus; in AV fistulae, it leaves for a vein. - Seen in tetralogy of Fallot.
- Can lead to volume overload on the LV and eccentric hypertrophy. - Described as mid-systolic murmur, or just regular “systolic” murmur, that
- Highest yield cause on USMLE is aortic dissection à can retrograde increases with inspiration, at the left sternal border, 2nd intercostal space.
propagate toward the aortic root causing aortic root dilatation and AR. Pulmonic stenosis
This is the theoretical location, whereas AS is the 2nd intercostal space on
- Even though MVP is most common in Marfan and Ehlers-Danlos, AR is the right, not left. But the USMLE often isn’t strict about murmur locations
second most common in these patients, since if they get aortic dissection, this way.
this can lead to AR. - Ductus arteriosus is special vessel in fetal circulation that connects the
- Peds shelf can give you random 2-year-old with robust decrescendo proximal pulmonary trunk to the descending arch of the aorta. This allows
diastolic murmur (answer = AR). Student says, “What’s causing it though?” for blood to bypass the high-resistance lungs in utero. After birth, this
Great question. It’s still what shelf will do. You could be aware that vessel should close, resulting in a remnant called the ligamentum
congenital bicuspid valve also can cause AR in peds, not just AS. arteriosum, but sometimes it does not close à PDA.
- Mid-systolic murmur, or just “systolic” murmur; can also be described as - If a PDA occurs, blood moves in the neonate LàR (i.e., opposite of in
“late-peaking systolic murmur with an ejection click.” utero) from the descending arch of the aorta to the pulmonary trunk.
- Radiates to the carotids. This descriptor shows up quite frequently on
NBME (way more than radiation to the axilla for MR). Patent ductus arteriosus
- Causes slow-rising pulses, aka “pulsus parvus et tardus” (don’t confuse (PDA)
with bounding pulses of AR).
Aortic stenosis - SAD à Syncope, Angina, Dyspnea; classic combination seen in AS, albeit
not mandatory. If you get a question where they say systolic murmur but
you’re not sure of the diagnosis, if they say chest pain or fainting, you
know it’s AS.
- Often caused by bicuspid aortic valve. The patient need not have Turner
syndrome and often won’t. Bicuspid valve is usually inherited as an
autosomal dominant familial condition.

MEHLMANMEDICAL.COM 6 MEHLMANMEDICAL.COM 7
MEHLMANMEDICAL.COM MEHLMANMEDICAL.COM

- You can see in the above diagram, somehow the blood become more - The child will not be cyanotic at birth, but then years later, will develop
oxygenated from the RV to the pulmonary artery, which is ordinarily Eisenmenger syndrome (i.e., a reversal of the LàR shunt over the VSD to
impossible. The only way this could have occurred is if oxygenated blood be RàL) and cyanosis, where the stem gives a school-age kid who squats
came LàR from the aorta to the pulmonary artery via a PDA. on the playground to relieve symptoms.
- Murmur described three ways on USMLE: 1) continuous, machinery-like - Squatting ­ afterload, which ­ LV pressure, which ¯ the pressure
murmur; 2) pan-systolic pan-diastolic murmur (meaning it’s continuous gradient of the RàL shunt, thereby mitigating cyanosis.
throughout both systolic and diastole); and 3) to-and-fro. The latter shows - Squatting also ­ preload by ­ venous return back to the right heart. But it
up on 2CK offline NBME 6. is the effect of ­ afterload that is most related to the ¯ in symptoms.
- Classically associated with congenital rubella (HY). They’ll give a kid born - Aka persistent fetal circulation.
with a PDA and then ask what the mom experienced while pregnant; - Q will give a post-term birth at 42 or 43 weeks + meconium-stained fluid
answer = arthritis and/or rash (rubella often presents as arthritis in adults). + echo of the neonate shows a RàL shunt across the foramen ovale.
- Indomethacin (NSAID) will close the PDA. Student says, “Wait, but isn’t the foramen ovale between the atria, and
- Prostaglandin E1 is used to keep a PDA open (if a kid with congenital Persistent fetal hypertension that’s only open in the fetus but is supposed to close after birth?” Correct.
heart malformations is born cyanotic and we need to buy time until Hence we have persistent fetal circulation.
surgery). - Answer on USMLE will be “failure of pulmonary vasodilation.” Meconium
- An open PDA can mask cyanosis in a newborn in a variety of conditions aspiration syndrome can ¯ opening of the lung vasculature, leading to ­
(i.e.,., hypoplastic left heart syndrome or pre-ductal coarctation). If they right heart pressure and ­ risk of persistent fetal circulation.
tell you a kid is born with normal APGAR scores but a week later becomes - SLE in pregnant women can cause congenital heart block in neonates.
cyanotic and they ask why, the answer is “closure of ductus arteriosus.” Congenital heart block
Sounds weird, but shows up on NBME somewhere.
- Systolic murmur seen in the setting of higher heart rate caused by - Idiopathic arrhythmia disorder that causes transient hypoxia to the
infection, anemia, or pregnancy. Caused by increased flow across the Adams-Stokes attack brainstem, resulting in seizure-like episodes.
pulmonic and/or aortic valves. - EEG does not show any seizure activity.
- Known as a functional murmur because this means it goes away once the
heart rate comes back down.
Functional (flow) murmur - Seen all over 2CK Peds forms, where they try to trick you into thinking the
kid has a valvular pathology of some kind, but there isn’t; there will merely
Rheumatic heart disease (rheumatic fever) HY points
be an infection or simple viral infection.
- Can be seen sometimes with ASD, where the patient will have fixed - Strep pyogenes (Group A Strep) oropharyngeal infection results in production of antibodies against S.
splitting of S2 “plus a systolic murmur” à merely higher right-sided pyogenes’ M-protein that cross-react with the mitral valve (i.e., molecular mimicry; type II hypersensitivity).
volume, so more flow across the pulmonic valve. - Can occur with the aortic valve in theory, but on USMLE, it is always mitral valve.
- Results in mitral regurgitation acutely and mitral stenosis late, as discussed earlier.
- On 2CK Peds form; described as a murmur in the neck that abates when
Venous hum the kid is laid supine + the neck rotated. - Presents as JONES (J©NES) à Joints (polyarthritis), © Carditis, subcutaneous Nodules, Erythema
- Benign + don’t treat. marginatum (annular, serpent-like rash), Sydenham chorea (autoimmune basal ganglia dysfunction that
results in dance-like movements of the limbs).
- Associated with cardiac tumors (i.e., myxoma in adult, or rhabdomyoma
- Cutaneous Group A Strep infections don’t cause rheumatic fever, but can still cause PSGN.
in kids for tuberous sclerosis).
“Ball-in-valve” murmur - Tx = penicillin.
- Described as a diastolic rumbling murmur that abates when the patient is
re-positioned unconventionally (e.g., onto his or her right side).
- Refers to narrowing of the aortic arch (this is referred to as coarctation;
do not use the word stenosis to describe this).
- Classically seen in Turner syndrome, but absolutely not mandatory.
Shows up idiopathically in plenty of NBME Qs. I point this out because
students often think the patient must have Turner syndrome.
- Presents as upper extremities that have higher BP, brisk pulses, and are
warmer; the lower extremities have lower BP, weak pulses, and are cooler.
Coarctation of the aorta - Sometimes the Q can just say, “the radial pulses are brisk.” à The
implication is, “Well if they’re saying specifically that the radial pulses are
brisk, that must mean the pulses in the legs aren’t.”
- Murmur sound not important for USMLE. Can sometimes be described as
a systolic murmur heart in the infrascapular region.
- Can cause LVH with left-axis deviation ECG (on NBME).
- USMLE doesn’t give a fuck about pre- vs post-ductal. Pre-ductal in theory Erythema marginatum
will be a very sick neonate. Post-ductal will be an adult (most cases).
- 1) Pulmonic stenosis; 2) RVH; 3) overriding aorta; 4) VSD.
Tetralogy of Fallot - If you’re asked which component most determines prognosis, the answer
is the degree of pulmonic stenosis.

MEHLMANMEDICAL.COM 8 MEHLMANMEDICAL.COM 9
MEHLMANMEDICAL.COM

Peds conditions confused for cardiac path


- Psych forms love trying to make you think this is an MI.
- They’ll give you a 17-yr-old male who feels doom / like he’s going to die.
- Sometimes they mention in stem Hx of MI in family as distraction.
- They can say patient has mid-systolic click and then they ask for cause of
Panic attack
patient’s symptoms à answer = panic disorder, not MVP. Student gets
confused, but MVP is almost always asymptomatic, where panic attack is
clearly cause of the patient effusively hyperventilating.
- Treat with benzo.
- Fainting in response to stressor (e.g., emotional trigger).
- Stress triggers an initial sympathetic response, which in turn triggers a
compensatory parasympathetic response. This latter response is excessive in
Vasovagal syncope some people, where the peripheral arterioles dilate and the heart slows too
much à decreased cerebral perfusion à lightheadedness/fainting.
- 2CK wants you to know a tilt-table test can be used to diagnose, where a
reproduction of symptoms can occur.
- Not cardiac. This is MSK. But I’m putting this here real quick because
students often confuse this for cardio.
- Inflammation of cartilage at rib joints.
Costochondritis - Will present as chest pain that worsens with palpation or when patient
reaches over the head or behind the back. These two findings are clear
indicators we have an MSK condition, not cardiac.
- Can be idiopathic, caused by strain (e.g., at the gym), or even post-viral.
- MSK condition asked twice on 2CK material (once on FM form; also on Free
120) that has nothing to do with the lungs, despite the name.
- This is viral infection (Coxsackie B) causing sharp lateral chest pain due to
Pleurodynia
intercostal muscle spasm. Sometimes students choose pericarditis, etc., even
though the presentations are completely disparate.
- Creatine kinase can be elevated in stem due to ­ tone of muscle.
- Viral infection causing inflammation of the pleura (layers covering the lungs),
leading to sharp chest pain.
Viral pleurisy
- If this is the answer, CK will be normal (unlike pleurodynia, because it’s not
MSK).
- They’ll say 3-year-old was having a tantrum followed by falling on the floor +
appearing blue.
- Child will involuntarily stop breathing following a trigger – e.g., being upset,
frightened, or experiencing pain.
Breath-holding spell
- The child often cries or becomes upset, exhales forcefully, stops breathing,
then develops cyanosis, which can sometimes be followed by brief loss of
consciousness and jerking (can be mistaken for seizure).
- Shows up on Peds forms so you need to know it exists.

MEHLMANMEDICAL.COM 10

You might also like